Chuyên đề bất đẳng thức từ tập thể trường THPT chuyên Lê Quý Đôn - Quảng Trị

124 13 0
Chuyên đề bất đẳng thức từ tập thể trường THPT chuyên Lê Quý Đôn - Quảng Trị

Đang tải... (xem toàn văn)

Tài liệu hạn chế xem trước, để xem đầy đủ mời bạn chọn Tải xuống

Thông tin tài liệu

Tuy nhiên phần lớn học sinh khi học môn Lượng giác học (giải phương trình lượng giác, hàm số lượng giác …), lại thấy nó như là một bộ phận của môn Đại số học, hoặc như một công cụ để [r]

(1)

N

M O

O1

O2 C

ha

x y z

N Q

P A

B C

M

n

n

n aa a

n a a

a

2

1   

2 cos cos

cosABC

R c b a z y x

2

2 2 

  

    2

2 2

2 2

2

1b a b anbn a a an b b bn

a          

xyz z y x z

C y

B x

A

2 cos

cos

cos  2

 

2 tan tan tan cot

cot cot

2 2

2 2

C B A

c b a C

B A

c b a

    

 

 

(2)

Lời mở đầu

“Nơi vật lý hố học dừng chân nơi toán học bắt đầu”

Toán học mang sự bao la phong phú vô tận khoa học tự nhiên Toán học bầu trời đêm thăm thẳm đầy lấp lánh Một sáng mang tên “Bất đẳng thức”

Bất đẳng thức lĩnh vực đặc sắc Đây kết hợp hoàn hảo Đại số Hình học Một vấn đề mang lại bao hứng thú cho nhà toán học, cho giáo viên dạy toán, cho học sinh giỏi toán khắp nơi Tất mang nét quyến rũ bí ẩn đặc trưng tốn học Vì vấn đề hấp dẫn đề tài nghiên cứu khám phá cho hệ người học toán khứ, tương lai

Đọc đến có lẽ bạn đọc cho tác giả lời Nhưng ! Sau đọc chuyên đề này, bạn đọc đồng ý với tác giả Chuyên đề “Bất đẳng thức” đưa từ bất đẳng thức dễ chứng minh đến tốn gay go phức tạp, từ phương pháp cở điển quen thuộc đến phương pháp đại mẻ Vì chuyên đề phù hợp cho trình độ người đọc

Chuyên đề “Bất đẳng thức ” chia làm chương : Chương 1: Các bước đầu sở

Chương này tác giả trang bi ̣ cho người đo ̣c những “vâ ̣t du ̣ng” cần thiết cho việc chứng minh bất đẳng thức

Chương 2: Các phương pháp chứng minh.

Chương sẽ bao gồ m hầu toàn bô ̣ các phương pháp thường dùng chứ ng minh bất đẳng thức

Chương 3: Áp dụng vào một số vấn đề khác

Các bất đẳng thứ c đươ ̣c vâ ̣n du ̣ng để giải quyết mô ̣t số vấn đề khác giải phương trình, ̣nh tính tam giác, …

Chương 4: Một số chuyên đề, bài viết hay, thú vi ̣ liên quan đến bất đẳng thức.

Chương 5: Bất đẳng thức thế nào là hay ?

Là m có thể sáng tạo bất đẳng thức?

Đây lại là mô ̣t chương thú vi ̣ về quan niê ̣m bất đẳng thức của tác giả và mô ̣t số ý kiến quan điểm của giáo viên toán, ho ̣c sinh giỏi toán quen thân vớ i tác giả đươ ̣c thu thâ ̣p và trình bày

(3)

Mong rằ ng chuyên đề “Bất đẳng thức” sẽ trở thành người ba ̣n đồng hành đường khám phá vẻ đe ̣p “Toán học muôn màu” củ a ba ̣n đo ̣c

Cuố i cù ng chân thành gửi lời cảm ơn đến các ba ̣n HS chuyên toán khóa 2008 – 2011 Trườ ng THPT chuyên Lê Quý Đôn, Quảng Trị đã ủng hộ hỗ trợ giúp cho chuyên đề trở nên phong phú đa da ̣ng

Và chân thành cảm ơn cựu học sinh chuyên toán:

- Trương Hữu Hà Ninh (HS chuyên Toá n khóa 2002 – 2005 Trường THPT chuyên Lê Quý Đôn, Quảng Trị )

- Trương Hữu Đơng Hà (HS chun Tốn khóa 2000-2003 Trường THPT chuyên Lê Quý Đôn, Quảng Trị)

Và thầy giáo:

- Nguyễn Văn Hiền (GV chuyên toá n Trường THPT chuyên Lê Quý Đôn, Quảng Trị )

Đã giúp đỡ, đóng góp ý kiến để chuyên đề tốt

Quảng Trị, ngày 25 tháng 02 năm 2009

HS tổ 4, lớp chuyên toá n khóa 2008 – 2011 Trườ ng THPT chuyên Lê Quý Đôn, Quảng Trị

Mọi thắc mắc, ý kiến đóng góp về chuyên đề “Bất đẳng thức ” xin gử i cho tác giả theo email : truonggiang250293@yahoo.com hay nick

truonggiang250293 www.diendantoanhoc.net,

www.mathnfriend.net và www.diendan3t.net

(4)

Trong chuyên đề này, ta dùng gần xuyên suốt các ký hiê ̣u sau :

ABC

 : tam giác ABC

C B

A, , : các góc của tam giác ABC

c b

a, , : các ca ̣nh đố i diê ̣n lần lươ ̣t với các góc A,B,C

c b a h h

h , , : các đường cao ứng với các ca ̣nh

c b a m m

m , , : các đường trung tuyến ứng với các ca ̣nh

c b a l l

l , , : các đường phân giác ứng với các góc

S R r

p, , , nử a chu vi , bán kính nô ̣i tiếp, bán kính ngoa ̣i tiếp, diê ̣n tích tam giác ABC

c b a r r

r , , bán kính đường tròn bàng tiếp ứng với các góc CMR : chứ ng minh rằng

Đpcm : điều phải chứng minh BĐT: bất đẳng thức

VP: vế phải VT: vế trái

 : suy

: tương đương

(5)

Chương :

CÁC BƯỚC ĐẦU CƠ SỞ

Để bắt đầu hành trình, ta khơng thể khơng chuẩn bị hành trang để lên đường Toán học Muốn khám phá hay đẹp bất đẳng thức, ta cần có “vật dụng” chắn hữu dụng, chương 1: “Các bước đầu sở”

Chương tổng quát kiến thức cần có để chứng minh bất đẳng thức Theo kinh nghiệm cá nhân mình, tác giả cho kiến thức đầy đủ cho “hành trình”

Trước hết bất đẳng thức đại số ( AM – GM, BCS, Jensen, Nesbitt,…) Tiếp theo đẳng thức, bất đẳng thức liên quan tam giác Cuối số định lý khác công cụ đắc lực việc chứng minh bất đẳng thức

Mục lục :

1.1 Các bất đẳng thức đại số 1.1.1 Bất đẳng thức Cauchy

1.1.1.1 Kĩ thuật chọn điểm rơi BĐT Cauchy 1.1.1.2 Kĩ thuật Cauchy ngược dấu

1.1.2 Bất đẳng thức BunhiaCốpxki

1.1.2.1 Kĩ thuật chọn điểm rơi BĐT BunhiaCốpxki 1.1.3 Bất đẳng thức Jensen

1.1.4 Bất đẳng thức Nesbitt

1.2 Các đẳng thức, bất đẳng thức tam giác 1.2.1 Đẳng thứ c

1.2.2 Bất đẳng thứ c

1.3 Bất đẳng thức đối xứng ba biến 1.3.1 Bất đẳng thức có điều kiện

(6)

1.1 Cá c bất đẳng thức bản : 1.1.1 Bất đẳng thứ c Cauchy :

Vớ i mo ̣i số thực không âm a1,a2, ,an ta có :

n n n a a a n a a a 2

1    

Bất đẳng thứ c Cauchy là một bất đẳng thức quen thuộc và có ứng dụng rất rộng rãi Đây là bất đẳng thức mà bạn đọc cần ghi nhớ rõ ràng nhất, nó sẽ là công cụ hoàn hảo cho viê ̣c chứng minh các bất đẳng thức Sau là hai cá ch chứng minh bất đẳng thức này mà theo ý kiến chủ quan của mình, tá c giả cho rằng là ngắn gọn và hay nhất

Chứ ng minh :

Cách : Quy nạp

Vớ i n1 bất đẳng thứ c hiển nhiên đúng Khi n2 bất đẳng thứ c trở thành

  2 2

1    

a a a a a a

(đú ng!) Giả sử bất đẳng thức đúng đến nk tứ c là :

k k k a a a k a a a 2

1   

Ta sẽ chứng minh nó đúng với n2k Thật vâ ̣y ta có :

          k k k k k k k k k k k k k k k k k k a a a a a k a a a k a a a k k a a a a a a k a a a a a a 2 2 2 2 2 2                       

Tiếp theo ta sẽ chứng minh với nk1 Khi đó :

 

1 1 1 1 1 1 1                        k k k k k k k k k k k k a a a k a a a a a a k a a a a a a k a a a a a a

(7)

Cách : ( lờ i giải của Polya )

Gọi

n a a

a

A     n

Khi đó bất đẳng thức cần chứng minh tương đương với n

n A

a a

a1 2  (*)

Rõ ràng nếu a1 a2  anA thì (*) có dấu đẳng thức Giả sử chúng không bằ ng Như vậy phải có ít nhất mô ̣t số, giả sử là a1  A và mô ̣t số khác, giả sử là a2  A tứ c là a1  Aa2

Trong tích Pa1a2 an ta hãy thay a1 bở i a'1 A và thay a2 bở i

A a a

a'2  1 2  Như vâ ̣y a'1a'2a1a2 mà

    

'

'1a2a2a2  A a1 a2 Aa1a2  a1A a2 A

a

2 ' ' a a a

a

n n a a a a

a a a

a1  '1 '2

Trong tích P'a'1a'2a3 an có thêm thừa số bằng A Nếu P' cò n

thừ a số khác A thì ta tiếp tu ̣c biến đổi để có thêm mô ̣t thừa số nữa bằng A Tiếp tục vâ ̣y tối đa n1 lần biến đổi ta đã thay mo ̣i thừa số P bằng A

và đươ ̣c tích n

A Vì quá trình biến đổi tích các thừa số tăng dần

n

A P

  đpcm

Ví du ̣ 1:

Cho A,B,C là ba góc của một tam giác nhọn CMR :

tanAtanBtanC 3

Lờ i giải :

Vì   C

B A

B A

C B

A tan

tan tan

tan tan

tan

tan 

  

  

tanAtanBtanCtanAtanBtanC

Tam giác ABC nho ̣n nên tanA,tanB,tanC dương Theo Cauchy ta có :

   

3 tan tan

tan

tan tan

tan 27 tan

tan tan

tan tan

tan tan tan tan tan tan

tan

2

3

 

 

 

 

 

 

 

 

C B

A

C B

A C

B A

C B

A C

B A C

B A

(8)

Ví dụ 2:

Cho a,b,c số d-ơng Chứng minh rằng:

2

2

3 3 3

c b a ca

a c bc

c b ab

b a

       

(1.1)

Lờ i giải :

Ta sÏ sư dơng B§T Cauchy nh- sau:

Ta cã a3+b3  a2b + ab2  a3+b3  ab(a+b) 

ab b a

2 3 

b a

; T-¬ng tù, ta còng cã:

2

, 2

3 3

3

a c ca

a c c b bc

c

b      

Cộng theo vế BĐT lại với ta đ-ợc BĐT cần chứng minh

Vớ d 3:Cho a, b, c số d-ơng Chứng minh r»ng:

3 13 3 13 3 13

abc abc a

c abc c

b abc b

a         

Lời giải:

Ta cã: a3+b3  a2b + ab2  a3+b3 + abc  a2b + ab2 +abc  a3+b3 + abc  ab(a+b+c)

, ) (

) (

1

3

c b a abc

c c

b a ab abc b

a        

T-¬ng tù , ta cã:

; ) (

1 ,

) (

1

3 3

3

c b a abc

b abc

a c c b a abc

a abc

c

b          

(9)

1.1.1.1 Kĩ thuật chọn điểm rơi BĐT Cauchy

Trong chứng minh bất đẳng thức, việc ghép sử dụng bất

đẳng thức sở không thuận lợi dễ dàng Khi sử dụng liên tiếp nhiều bất đẳng thức ta phải ý tời điều kiện để bất đẳng thức xảy ra, để điều kiện thỏa mãn suốt trình ta sử dụng bất đẳng thức tring gian Và bất đẳng thức Cauchy bất đẳng thức Để thấy kĩ thuật ta vào số ví dụ sau:

Ví dụ :Cho a3.Tìm giá trị nhỏ biểu thức S=a+

a

1

Phân tích tìm tịi lời giải *Xét bảng biến thiên a,

a

1

S để dự đoán Min S

a 10 11 12 … 30

a

1

4

5

6

7

8

9

10

11

12

1 … 30

1

S

3

4

5

6

7

8

9

10

10

11

11

12

12

… 30

30

Nhìn lại bảng biến thiên ta thấy a tăng S lớn từ dẵn đến việc dự đốn a=3 S nhận giá trị nhỏ nhất.Để dễ hiểu tạo ấn tượng ta nói Min S=

3 10

đạt “Điểm rơi : a=3”

Do bất đẳng thức côsi xảy dấu điều kiện số tham gia phải ,nên “Điểm rơi:a=3”ta sử dụng bất đẳng thức côsi trực tiếp cho số a

a

1

3

3

Lúc ta giả định sử dụng bất đẳng thức côsi cho cặp số 

    

a

a

,

 cho “điểm rơi:a=3”thì a

a

 tức ta có

lược đồ “điểm rơi” sau đây: Sơ đồ:

 

3

a

 

 3

1  

3 1

a

Từ ta biến đổi theo sơ đồ “Điểm rơi”được nêu *Lời giải: S=a+

a

1

=

     

a

a

9 +

8a

2

a

a

9 + 8

=

3 10

Vậy với a=3 Min S=

3 10

(10)

Ví dụ 2:Cho a2.Tìm giá trị nhỏ biểu thức S=a+ 12

a

*Sơ đồ điểm rơi:

 

2

a

 

2

a

 

1  

4 1

2 

a

*Lời giải:S =a+ 12

a = 

     

2

8 a

a a

+

8 6a

33

2

8 a

a a

 +

8 6

=

4

Với a=2 Min S=

4

Ví dụ : Cho a6.Tìm giá trị nhỏ biểu thức S=a2 +

a

18

*Sơ đồ điểm rơi :

 

36

a

 

 36

18  

6 18 18

a

*Lời giải :S=a2

+

a

18

= 

     

a

a 18

6

2

+ 

  

  

6

1

1 a2 

a

a 18

6

2

 + 

  

  

6

1

1 a2

=6

a a

+

6

1 a

  

  

6

6

1

6

   

 

 =36 +3

Vậy với a=6 Min S=2a+3

a=2

8

 

(11)

Vi dụ 4: Cho 0<a

1 .Tìm giá trị nhỏ biểu thức S=2a+

a

*Sơ đồ điểm rơi :

a=

2

 

 

 

4

2 

a

*Lời giải : S=2a + 12

a = 

 

  

2

1

a a

a + 2

8

a

3

2

1

a a

a  + 2

8

a

= 2

8

a

8 3 

 =5 Với a=

2

MinS=5

a,b >0

Ví dụ 5:Cho .Tìm giá trị nhỏ S=ab+ ab

1

a+b1

*Phân tích tìm tịi lời giải :Biểu thức S chứa biến số a,b

đặt t=ab t=

ab

1

thì S=t+

t

1

biểu thức chứa biến số Khi đổi biến số ta cần phải tìm miền xác định cho biến số mới,cụ thể là:

Đặt t= 

ab

1

ab=

t

1

t= 2 2

2 1

2 1

              

b a

ab =4

Bài tốn trở thành:Cho t4.Tìm giá trị nhỏ S=t+

t

1

*Lời giải :S=t+

6 17 16

4 15 16 15 16 15 16 16 15 16

 

           

  

t t

t t t

t t t

Với t=4 hay a=b=

2

MinS=

4 17

a=

2

8

(12)

Ví dụ 6: Cho a,b >0.Tìm giá trị nhỏ biểu thức S=

b a

ab ab

b a

  

*Phân tích tìm tịi lời giải:Do S biểu thức đối xứng với a,b nên dự đoán MinS đạt a=b>0

*Sơ đồ điểm rơi

  

2 

 

a a ab

b a

   

 2

2

2 

a

a b a

ab

*Lời giải :

S=    

ab b a b a

ab ab

b a ab

b a b

a ab ab

b a b a

ab ab

b a

4

     

     

 

      

=1+

2

 Với a=b>0 Min S=

2

Ví dụ 7: a,b,c>0

Cho Tìm giá trị nhỏ S=a+b+c+

c b a

1 1

  a+b+c

2

*Phân tích tìm tịi lời giải:Do S biểu thức đối xưng với a,b nên dự đoán MinS đạt a=b=c =

2

*Sơ đồ điêm rơi:

a=b=c=

2

   

 2

1 1

a b c

   

a=b  4

a=b=c=

2

4

(13)

3

6

4 1 4

1

1

1

6

abc c

b a c

b a

abc   

  

 

  

   

3

c b

a 

  

=3+

2 15 27

4

27    

  

c b

a

Với a=b=c=

2

thì MinS=

(14)

1.1.1.2 Kĩ thuật Cauchy ngược dấu:

Kĩ thuật Cô si ngược dấu kĩ thuật hay khéo léo, mới mẻ ấn tượng bất đẳng thức Cauchy Để thấy điều bạn Trần Tiến Minh thực phần với ví dụ sau:

Ví dụ 1: Các số dương a,b,c thỏa mản điều kiện a+b+c=0.Chứng minh bất đẳng thức:

2

1

1    a2  c c

b b

a

Lời giải:

Ta giải cách dùng BĐT cô si với mẩu số đẳng sau sẻ đổi chiều

2 2

1

1        ac c b b a a c c

b b

a (điều trái với giả thiết ?)

Ta giai cách khác 2

1 b a

 a1 2

2

2

ab a b ab a b

ab    

Từ BĐT trên,xây dựng hai BĐT tương tự với b,c cộng ba BĐT lại suy ra:

2

1

1 2 

          

ca bc ab c b a a c c

b b

a

Vì ab+bc+ca3 Đẳng thức xảy a=b=c

Ví dụ : Chứng minh với a,b,c,d số thực dương có tổng ta có BĐT:

1

1

1      a2  d d

c c

b b

a

Lời giải:

Tương tự ta có

2

1

2

2

ab a b ab a b ab a b a

      

(15)

2

1

1

1 2 2 

               da cd bc ab d c b a a d d c c b b a

vì ta có ab+bc+cd+da4.Đẳng thức xảy raa=b=c=d

Ví dụ 3: Chứng minh với số thực dương a,b,c,d ta ln có:

2 2 2 2 d c b a a d d d c c c b b b a

a   

        Lời giải:

Sử dụng BĐT si với hai số dương ta có 2 2 2 2 b a ab ab a b a ab a b a

a    

  

Tương tự ta có

2 ;

2 ;

2 2

3 2 2 d d a d d c c d c c b b c b b         

Cộng vế theo vế BĐT ta có

( ) 2 2 2 2 đpcm d c b a a d d d c c c b b b a

a    

      

Đẳng thức xảy a=b=c=d

Tương tự ta có tốn

Ví dụ 4: Chứng minh với số thực a,b,c,d ta ln có:

2

2 3

4 3 3 3 d c b a a d d d c c c b b b a

a   

       

Giải ví dụ 3:

Sử dụng BĐT cô si cho số dương ta có 3 2 2 3 3 3 b a ab ab a b b a ab a b a a         

Tương tự ta có 2 3 c b c b

b  

 ;

2 3 d c d c

c  

 ;

2 3 a d a d

d  

(16)

Đẳng thức xảy raa=b=c=d

Ví dụ 5: Chứng minh với số dương a,b,c,d thỏa mản điều kiện a+b+c+d=4 ta có:

1

1

1      ab2  d da c cd b bc a Lời giải:

Theo BĐT cô si cho hai số dương ta có:

2 1 2 2 c ab a c b c ab a c b c ab a bc a         ) ( 1 ) (

2 a ab abc

c b a ac a b a c a a b

a   

      

Hồn tồn tương tự ta có BĐT sau bcd bc b d c

b   

 ;

cda cd c d c

c   

 ;

dab da d a d

d   

Cộng vế theo vế BĐT ta có

      

a b

d a d c d c b c b a 2 2 1 1 ) ( dab cda bcd abc da cd bc ab d c b

a          

Từ BĐT cô si dể dàng chứng minh BĐT :

( )

4

1    

  

bc cd da a b c d ab

( )

16

1    

  

bcd cda dab a b c d abc

Do

1 1 1 1a2babcd2422 d a d c d c b c b a

Đẳng thức xảy raa=b=c=d=1

( ) ( ) ( )

3

2 23

3           

b c ab bc ca

a (ab)23 (bc)23 (ca)23 3

(17)

1.1.2 Bất đẳng thứ c Bunhia Cốpxki :

Vớ i hai bô ̣ số a1,a2, ,an và b1,b2, ,bn ta có :

    22 2

2 2

2 2

2

1b a b anbn a a an b b bn

a          

Nế u Cauchy là “cánh chim đầu đàn” viê ̣c chứng minh bất đẳng thức thì Bunhia Cốpxki lại là “cánh tay phải” hết sức đắc lực Với

Cauchy ta phải chú ý điều kiê ̣n các biến là không âm, đối với

Bunhia Cốpxki các biến không bi ̣ ràng buộc bởi điều kiê ̣n đó, chỉ cần là số

thực cũng đúng Chứng minh bất đẳng thức này cũng rất đơn giản

Chứ ng minh : Cách : Xét tam thức :

   2  2

2

1

)

(x a x b a x b anx bn

f       

Sau khai triển ta có :

     2

2 2

2 1 2

2

1

)

(x a a an x ab a b anbn x b b bn

f            

Mặt khác vì f(x)0xR nên :

              

 

 2

2 2

2 2

2

1

0 n n n n

f ab a b a b a a a b b b

đpcm

Đẳng thứ c xảy

n n

b a b

a b a

  

2

1 (quy ươ

́ c nếu bi 0 thì ai 0) Cách :

Sử du ̣ng bất đẳng thức Cauchy ta có :

  2

2 2

2 2

2 2

2

2 2

2

2

n n

i i n

i n

i

b b

b a a

a

b a b

b b

b a

a a

a

   

  

     

Cho i chạy từ đến n rồi cô ̣ng vế cả n bất đẳng thức la ̣i ta có đpcm Đây cũng là cách chứng minh hết sức ngắn gọn mà bạn đọc nên ghi nhớ! Bây giờ với sự tiếp sức của bất đẳng thức Bunhia Cốpxki , Cauchy được tiếp thêm nguồn sức mạnh, hổ mọc thêm cánh, rồng mọc thêm vây, phá t huy hiê ̣u quả tầm ảnh hưởng của mình Hai bất đẳng thức này bù đắp bổ sung hỗ trợ cho viê ̣c chứng minh bất đẳng thức Chúng đã “lưỡng long nhất thể”, “song kiếm hợp bích” công phá thành công nhiều bà i toán khó

(18)

Ví du ̣ 1: Cho a,b,c0 và asinxbcosyc CMR : 3 3 2 1 sin cos b a c b a b y a x     

Lờ i giải :

Bất đẳng thứ c cần chứng minh tương đương với :  * cos sin 1 cos sin 3 2 3 2 b a c b y a x b a c b a b y a x           

Theo Bunhia Cốpxki thì :

    2 2 2 2 2

1b a b a a b b

a    

vớ i

         b b b a a b b y a a x a 2 ; cos ; sin

 3  2

2 cos sin cos sin y b x a b a b y a

x   

       

 0

b

a và asinxbcosyc * đú ng  đpcm Đẳng thứ c xảy 2 2

2

1 sin cos

b y a x b a b a                        3 3 2 cos sin cos sin cos sin b a c b y b a c a x c y b x a b y a x

Ví du ̣

Chứ ng minh rằng :

(19)

Áp dụng bất đẳng thức BCS liên tiếp lần ta có :

    

    

4

2

2 2 2

2

2

8 sin

cos

8 sin

cos 1 1

sin cos 1 sin

cos

 

 

 

 

 

x x

x x

x x x

x

Đẳng thứ c xảy và chỉ

4  

(20)

1.1.2.1 Kĩ thuật chọn điểm rơi bất đẳng thức Bunhiacốpxki

Cũng bất đẳng thức Cauchy bất đẳng thức cần có phương pháp để cân hệ số ta giải toán liên quan đến bất đẳng thức

.Bất đẳng thức Bunhiacốpski

*Dạng 1:    1 2 2

2 2 2 2

1 a an b b bn a b a b anbn

a         

*Dạng 2: a a an b b bna1b1 a2b2 anbn

2 2 2 2

1        

*Dạng 3: aa  an  bb  bna1b1 a2b2  anbn

2 2 2 2

Dấu bằng: Dạng 1, dạng

n n b a b a b a     2 1

;dạng

2

1    

n n b a b a b a a,b,c>0

Ví dụ 1:Cho

.Tìm giá trị nhỏ biểu thức:

a+b+c6

S= 2

2 2

2 1

a c c b b

a     

*Phân tích tìm tịi lời giải:Xét dang đặc biệt với n=2:

  1 2 2 2 2 ]

[aa bbaba b Dấu xẩy 2

1  

b a b a

*Ý nghĩa:Chuyển đổi biểu thức thành biểu thức khác Xét đánh giá giả định vói số α, β

                              b a b a b

a  

  

 

 2

2 2 2 2

2 1 1

(1) +                               a b c b c

b  

  

 

 2

2 2 2 2

2 1 1

(2)                             a c a c a

c  

  

 

 2

2 2 2

2 1 1

(3)

 1

) ( S c b a

(21)

Do S biểu thức đối xứng với a, b, c nên dự đốn S=So điểm rơi

a=b=c=2, tất bất đẳng thức (1), (2), (3) đồng thơi xảy dấu tức ta có sơ đồ điểm rơi sau:

b b a   *Sơ đồ: a=b=c=2 

c b     1

1   

a c c b b a    a c   

Kết hợp với biến đổi theo “kỹ thuật điểm rơi cối ” ta có lơi giải sau:

*Lời giải đúng:

                  b a b a b

a

17 ) ( 17

1 2

2 2 +                   c b c b c

b

17 ) ( 17

1 2

2 2                   a c a c a

c

17 ) ( 17

1 2

2 2

2

S

                                c b a c b a c b a c b a c b

a 1

4 4 ) ( 15 17 1 1 4 17 17 3 45 17 1 1 4 6 15 17                        c b a c b a

Với a=b=c=2 Min S=

2 17

a,b,c >

Ví dụ 2: Cho Tìm Min S=

b a c a c b c b a      

 2

2

abc6

(22)

Bình luận lời giải

*Phân tích để tìm lời giải: Xét đánh giá giả định với số ,

  c b a c b a                   

 2 2  

2 (1) +   a c b a c b                   

 2 2  

2 (2)   b a c b a c                   

 2 2  

2

2 (3)

_                  a c c b b a c b a

S ( ) 1

2   

   So

a c c b b a c b a

S 

                   

 1 1

2

2   

Do biểu thức đối xứng với a,b,c nên dứ đốn S=So điểm rơi a=b=c=2,

đó bất dẳng thức (1), (2), (3) dồng thời xảy dáu tức có sơ đồ điểm rơi sau đây:

*Sơ đồ điểm rơi:

b a   

a=b=c=2        

1 1 1 a c c b b a b b     a c   

Từ ta có lời giải sau đây:

(23)

*Lời giải đúng: c b a c b a                   

 (42 12) 2 + a c b a c b                   

 (42 12) 2 b a b b a c                   

 (42 12) 2                 a c c b b a c b a

S 4( ) 1

17 a c c b b a c b a a c c b b a c b a               

 4( )

) (      

  6( )

9 ) ( ) 1 ( ) (

3 2 a b c a b c

a c c b b a c b a                  ) ( ) ( ) ( 31 c b a c b a c b a c b a             51 93 ) ( ) ( 8 31

3   

          c b a c b a c b a 17 17 17 17

51  

S Với a=b=c=2 S=

2 17

Ví dụ3: Cho a, b, c > thoả mãn a+b+c+ 2abc10. Chứng minh

S= 6

4 9

8 2

2 2 2 2

2         

c b a c b a c b a c b a Lời giải:

Dự đoán điểm rơi: a = b = c =

(24)

ca b a a c b

a     

4 18 2 2

+ c ab

b b a c

b     

4 18 2 2 bc a c c b a

c     

4 18 2 2 _           c b a

S 1

24 +9(a+b+c)+ab+bc+ca ) ( 2 4 ) ( ) ( ) ( ) ( 4 c b a abc abc abc c c b b a a c b a ab c ca bb bc a c c b b a a                                              6 24 / 72 72 10 12 ) (

12         

(25)

1.1.3 Bất đẳng thứ c Jensen :

Hàm số yf(x) liên tục đoa ̣n  a,b và n điểm x1,x2, ,xn tù y ý

đoa ̣n  a,b ta có :

i) f ''(x)0 khoả ng  a,b thì :

  

   

 

 

n x x

x nf x f x

f x

f n

n

)

( ) ( )

(

2

ii) f ''(x)0 khoả ng  a,b thì :

  

   

 

 

n x x

x nf x f x

f x

f n n

)

( ) ( )

( 2

Bất đẳng thứ c Cauchy và bất đẳng thức Bunhia Cốpxki thật sự là các đại

gia việc chứng minh bất đẳng thức nói chung Nhưng riêng đối với chuyên mục bất đẳng thức lượng giác thì đó lại trở thành sân chơi riêng cho bất đẳng thứ c Jensen Dù có vẻ khó tin đó là sự thật, đến 75% bất đẳng thứ c lượng giác ta chỉ cần nói “theo bất đẳng thức Jensen hiển nhiên ta có đpcm”

Cho f :R R thỏ a mãn     

     

f y f x y x y R x

f ,

2 ) ( )

( Khi đó với

mọi x1,x2, ,xnR ta có bất đẳng thức :

  

   

 

 

n x x

x nf x f x

f x

f n n

)

( ) ( )

( 2

̣ thật là tác giả chưa từng tiếp xúc với một chứng minh chính thức của

bất đẳng thứ c Jensen phát biểu có f ''(x) Cò n viê ̣c chứng minh phát biểu thì rất đơn giản Nó sử dụng phương pháp quy nạp Cauchy Do đó sẽ không trình bày chứng minh ở

(26)

Ví du ̣

Chứ ng minh rằng với mọi ABC ta có :

2 3 sin sin

sinABC

Lờ i giải :

Xét f(x)sinx vớ i x0;

Ta có f ''(x)sinx0x0; Từ đó theo Jensen thì :         

  

   

 

2 3 sin 3

3f A B C

C f B f A

f đpcm

Đẳng thứ c xảy và chỉ ABC đều

Ví du ̣

Chứ ng minh rằng với mọi ABC đều ta có :

2 tan tan

tan ABC

Lờ i giải :

Xét f x tanx vớ i       

2 ; 

x

Ta có   

        

2 ; 0

cos sin '

' 3 x

x x x

f Từ đó theo Jensen thì :

  

      

   

   

                   

3 sin 3

2 2 2

2

C

B A f C f B f A

f đpcm

(27)

1.2 Cá c đẳng thức bất đẳng thức tam giác :

Sau là hầu hết những đẳng thức, bất đẳng thức quen thuộc tam

giá c và lượng giác được dùng chuyên đề này hoặc rất cần thiết cho quá trình học toán của bạn đọc Ta có thể dùng phần này một từ điển nhỏ để tra cứu cần thiết.Hay cũng có thể chứng minh tất cả các kết quả là bài tập rèn luyê ̣n Ngoài cũng xin nhắc với bạn đọc rằng những kiến thức phần này áp dụng vào bài tập đều cần thiết được chứ ng minh lại

1.2.1 Đẳng thứ c :

R C c B b A a sin sin

sin   

C ab b a c B ca a c b A bc c b a cos cos cos 2 2 2 2 2          A b B a c C a A c b B c C b a cos cos cos cos cos cos            

p ap bp c

p r c p r b p r a p pr C B A R R abc C ab B ca A bc h c h b h a S c b a c b a                    sin sin sin sin sin sin 2 2 2 2 2 2 2 2 2 2 c b a m b a c m a c b m c b a          b a C ab l a c B ca l c b A bc l c b a       cos 2 cos 2 cos

 tan  tan  tan sin sin sin

2 2 2

A B C A B C

(28)

                                                   tan tan tan tan tan tan A C A C a c a c C B C B c b c b B A B A b a b a S c b a C B A S c b a C S b a c B S a c b A cot cot cot cot cot cot 2 2 2 2 2 2                        ab b p a p C ca a p c p B bc c p b p A          sin sin sin       ab c p p C ca b p p B bc a p p A       cos cos cos             

p c

(29)(30)

1.3 Bất đẳng thức đối xứng ba biến

Bất đẳng thức đối xứng phần quan trọng bất đẳng thức sơ cấp , yêu thích khơng với bạn thành thạo mà hấp dẫn với bạn bắt đầu Có lẽ lí đơn giản bất đẳng thức dạng đẹp chuẩn hình thức

Dạng tổng quát :

f ( a,b,c) ≥

Trong f (a,b,c) hàm đối xứng biến a,b,c hay nói cách khác f (a,b,c) = f (c,b,a) = f (b,a,c) Chẳng hạn : f (a,b,c) = a2 + b2 + c2 + 3ab + 3bc + 3ca +5abc + a2b2c2

Tính chất quan trọng biểu thức đối xứng vai trị bình đẳng giữa biến ,và ta xếp lại theo trật tự tuỳ ý giá trị biến số chứng minh Các tính chất định nghĩa mở rộng tương tự với biểu thức n biến x1,x2,x3,…xn.

1.3.1. Bất đẳng thức nhât khơng có điều kiện

Hàm f (a,b,c) gọi với biến miền I

thỏa mãn điều kiện

f (ta,tb,tc) = tk f (a,b,c)

với t,a,b,c Є I k số không phụ thuộc vào a,b,c,t mà phụ thuộc vào thân hàm f Trong phạm vi đa thức đa thức thuần tổng đơn thức đồng bậc

Chẳng hạn : f (a,b,c) = a5bc3 + a2b3c4 + ab6c2 đa thức

không đối xứng

(31)

Bất đẳng thức liên quan : Bất đẳng thức Cauchy :

Với số thực dương a1,a2,…,an có bất đẳng thức:

n

a a

a1  2   nn

n

a a a1 2 Một số ví dụ:

Ví dụ 1: (Bất đẳng thức Schur) Với số thực không âm a,b,c ta ln có bất đẳng thức :

a3 + b3 + c3 + 3abcab(a + b) + bc(b + c) + ca(c + a) (1) Lời giải :

Do tính đối xứng bất đẳng thức ta giả sử a ≥ b ≥ c Đặt x = a − b ,y = b − c ,khi (1) trở thành:

c(x + y)y − (c + y)xy + (c + x + y)x(x + y) ≥

‹=› c(x2 + xy +y2) + x2(x + 2y) ≥

Bất đẳng thức hiển nhiên biến c,x,y không âm Đẳng thức xảy x = y = x = c =0 hay a = b = c a = b, c =

1.3.2: Bất đẳng thức đối xứng có điều kiện :

Các bất đẳng thức đối xứng có điều kiện khơng có điều kiện đối tượng riêng rẽ tồn đọc lập thật lại có mối quan hệ chặt chẽ với nhau Sau số ví dụ:

Ví dụ 1: CMR với số thực a,b,c không âm :

3 ca bc ab 

8

) )( )(

(ab bc ca

Lời giải:

Giả sử ab + bc + ca = 3,khi a + b + c ≥ 3 abc ≤ 1 Mà (a + b)(b + c)(c + a) = (a + b + c)(ab + bc +ca) − abc = 3(a + b + c) − abc

=>

3

ca bc ab 

= ≤

8

) )( )(

(32)

Suy điều phải chứng minh.Dấu “=” xảy <=> a = b = c

Ví dụ 2: CMR với số a.b.c khơng âm ta ln có :

a2(b + c) + b2(c + a) + c2(a + b) ≥ (ab + bc + ca)3 (ab)(bc)(ca) Lời giải:

Giả sử (a + b)(b + c)(c + a) = , cần chứng minh :

a2(b + c) + b2(c + a) + c2(a + b) ≥ (ab + bc + ca) Tiếp chứng minh bất đẳng thức :

ab + bc + ca ≤ (1)

a2(b + c) + b2(c + a) + c2(a + b) ≥ (2) Thật ,với bất đẳng thức (1) ta có :

= (a + b + c)(ab + bc + ca) − abc => ab + bc + ca =

c b a

abc

 

8

Theo bất đẳng thức Cauchy :

= (a + b)(b + c)(c + a) ≥ 8abc => abc ≤ = (a + b)(b + c)(c + a) ≤ (

3 ) (

2 abc

)3

=> a + b + c ≥ Do : ab + bc + ca

3 )

( abc

a2(b + c) + b2(c + a) + c2(a + b)=(a + b)(b + c)(c + a) − 2abc = 8−2abc ≥ Suy đpcm

(33)

1.4 Bài tâ ̣p :

Bi 1: Cho a, b, c số d-¬ng Chøng minh r»ng:

3 2 2 c b a a ca c c c bc b b b ab a

a   

       

Bài 2: Cho a,b,c0 a+b+c=3 Chứng minh: 2 2 2 2     

c a

c c b b b a a

Bài 3: Cho a,b,c 0 a+b+c=3.Chứng minh rằng:

3

2 2b a

a

 2

2      a c c c b b

Bài 4: Chứng minh với số dương a,b,c,d có tổng

1 1 1 2

2  

       a c c b b a

Bài 5: Chứng minh với số dương a,b,c,d có tổng

1 1 1 1 2

2  

          a d d c c b b a

Bài 6: Chứng minh với a,b,c,d dưng có tổnh

1 1 1 1 2

2        

d c

b a

Bài 7: Cho a, b, c > 12 2

1

1 

                     c c b b a

a Chứng minh:

29 19 16 19 16 19

16

2 2

2            

b b a

c b a b a a c b a S

Bài 8: CMR với số thực a,b,c :

a6 + b6 + c6 + a2b2c2

(a5(b + c) + b5(c + a) + c5(b+c))

(34)

Chương :

Cá c phương pháp chứng minh

Chứ ng minh bất đẳng thức đòi hỏi kỹ và kinh nghiê ̣m Không thể khơi khơi mà ta đâm đầu vào chứng minh gă ̣p mô ̣t bài bất đẳng thức Ta sẽ xem xét nó thuô ̣c da ̣ng bài nào, nên dùng phương pháp nào để chứng minh Lú c đó viê ̣c chứng minh bất đẳng thức mới thành công đươ ̣c

Như vậy, để có thể đương đầu với các bất đẳng thức lượng giác, ba ̣n đo ̣c cần nắ m vững các phương pháp chứng minh Đó sẽ là kim chỉ nam cho các bài bất đẳng thức Những phương pháp đó cũng rất phong phú và đa da ̣ng : tổng hợp, phân tích, quy ước đúng, ước lượng non già, đổi biến, cho ̣n phần tử cực tri ̣ … Nhưng theo ý kiến chủ quan của mình, những phương pháp thâ ̣t sự cần thiết và thông du ̣ng sẽ được tác giả giới thiê ̣u chương : “Các phương pháp chứng minh”

Mục lu ̣c :

2.1 Biến đổi tương đương, tính chất bất đẳng thức 2.2 Sử du ̣ng các bước đầu sở

2.3 Đưa về vector và tích vô hướng 2.4 Phương pháp quy nạp

2.5 Phương pháp phản chứng

2.6 Phương pháp dùng tam thức bậc hai 2.7 Sử dụng số bất đẳng thức phụ 2.8 Sử dụng định lí Viét

(35)

2.1 Biến đổi tương đương :

Có thể nói phương pháp này là một phương pháp “xưa Trái Đất” Nó sử dụng các công thức và sự biến đổi qua lại giữa các bất đẳng thức Để có thể sử dụng tốt phương pháp này bạn đọc cần trang bi ̣ cho mình những kiến thứ c cần thiết

A Kiến thức cần nhớ: _ A   B A B

_ Ta biến đổi bất đẳng thức( BĐT) cần chứng minh tương đương với BĐT bất đẳng thức chứng minh

_ Chú ý đẳng thức sau:

 

 

  

2 2 2

2 2 2 2

3 3 2

2

2 2

3

a b a ab b

a b c a b c ab bc ca

a b c abc a b c a b c ab bc ca

   

       

          

_ Chú ý phép biến đổi tương đươngBĐT:

A    B A C B C

Với C>0: A B ACBC

Vơí C<0:A B ACBC

Với A,B>0 A B 1

A B

  

B Các ví dụ: C

I.Bài tốn có kèm với điều kiện:

Ví dụ 1: Chứng minh với x,y,z thoả mãn điều kiện 2

xyz ta

1

1 xy yz zx

    

Giải:

Ta cần chứng minh bất đẳng thức kép:

 2 2  2 2

1

2 x y z xy yz zx x y z

        

* Ta có:

 

 

2 2

2 2

2

2 2

0

x y z xy yz zx

a b c xy yz zx

x y z

     

      

(36)

BĐT nên ta có:

 2 2

1

2 x y z xy yz zx

     

* Mặt khác ta có:

 

 

     

2 2

2 2

2 2

2 2

0

xy yz zx x y z

xy yz zx x y z

x y y z z x

    

     

      

Bất đẳng thức nên ta có:  2 2

xyyzzxxyz

Như ta có điều phải chứng minh: 1 xy yz zx

    

Ví dụ 2: a,b,c ba số tuỳ ý thuộc đoạn [0;1] Chứng minh rằng:

2 2 2

1

abc  a b b c c a 

Lời giải: Nhận xét rằng:

1 1 1 

1

a b c

a b c ab bc ca abc

   

        

BĐT cần chứng minh tương đương với BĐT:

     

2 2

1 1

a  b b  c ca

Vì 0a b c, , 1nên 2

, ,

aa bb cc Do đó:

           

2 2

1 1 1

a  b b  c caa  b b  c ca

Vì ta cần chứng minh:

1  1  1 

a  b b  c ca 1

   

1

1 1

a b c ab bc ca

a b c abc

       

     

BĐT nên ta có điều phải chứng minh

(37)

II Bài tốn khơng có điều kiện:

Ví dụ 1:Chứng minh với a,b Ta có:

1

a b a b

a b a b

 

    Dấu xảy nào?

Lời giải: Ta có:

    

1

1

a b a b

a b a b

a b a b a b a b

a b a b

 

   

       

   

Bất đẳng thức Vây:

1

a b a b

a b a b

 

   

Ví dụ 2:Chứng minh với số dương a,b,c ta có:

3 3

2 2 2

3

a b c a b c

a ab b b bc c c ca a

 

  

     

Lời giải: Ta có:

  

  

3

2

3 2

3 2 2

3

3

0

a a b

a ab b

a a b a ab b

a b a b ab a b a b

   

    

    

   

Bất đẳng thức nên ta có: 2 2

3

a a b

a ab b

 

 

Tương tự ta có

3

2

3

2

2

a

b b c

b bc c

c c a

c ca

   

   

(38)

2.2 Sử du ̣ng các bước đầu sở :

Ta sẽ đưa các bất đẳng thức cần chứng minh về các bất đẳng thức bản

bắng cá ch biến đổi và sử dụng các đẳng thức bản Ngoài ra, tham gia cá c kỳ thi, tác giả khuyên bạn đọc nên chứng minh các đẳng thức, bất đẳng thứ c bản sử dụng một bổ đề cho bài toán

Ví du ̣

CMR mọi tam giác ta đều có :

2 sin sin sin 4 sin sin sin

sin sin

sinA BB CC A  A B C

Lờ i giải : Ta có :

2 sin sin sin cos cos

cosABC   A B C

Bất đẳng thứ c đã cho tương đương với :

cos cos cos  1

3 sin sin sin

sin sin

sinA BB CC A  ABC

mà :

B A B

A C

A C A

C B

C B C

B A

cos cos sin

sin cos

cos cos sin

sin cos

cos cos sin

sin cos

 

 

 

nên :

   2

4 cos cos cos

cos cos

cos

1  A BB CC A

Thật vâ ̣y hiển nhiên ta có :

cos cos cos   3

3 cos cos cos

cos cos

cosA BB CC AABC

Mặt khác ta có :

2 cos cos

cosABC

(39)

Ví du ̣

Cho ABC bất kỳ CMR :

cos cos cos

1 cos

cos cos

1 cos

cos cos

1

 

  

  

A A B B B C C C A

Lờ i giải :

Đặt vế trái bất đẳng thức cần chứng minh là T Theo Cauchy ta có :

T32cosAcosBcosC 4cosAcosBcosBcosCcosCcosA9  1 mà :

2 cos cos

cosABC

và hiển nhiên :

 

4 3

cos cos

cos cos

cos cos

cos cos

cos

2

 

 

B C C A A B C

B A

32cosAcosBcosC 4cosAcosBcosBcosCcosCcosA9  2

Từ    1, suy T 1đpcm

Ví du ̣

CMR vớ i mọi ABC bất kỳ , ta có :

2   2  2 2

4 S a b b c c a

c b

a         

Lờ i giải :

Bất đẳng thứ c cần chứng minh tương đương với :

2abbcca4 3Sa2b2 c2  1

Ta có :

S c b a C

S b a c B

S a c b A

4 cot

4 cot

4 cot

2 2

2 2

2 2

  

  

  

(40)

   

3 tan tan tan

3 cot

sin cot

sin cot

sin

cot cot

cot 4 sin

1 sin

1 sin

1

 

 

    

 

    

 

    

 

 

 

   

  

C B

A

C C

B B

A A

C B

A S S C

B A

S

đpcm

(41)

Tổ 4.vip.pro.friendly – lớp 10 Toán 41 A

e

e

2.3 Đưa về vectơ và tích vô hướng :

Phương phá p này đưa cho bạn đọc những lời giải bất ngờ và thú vị Nó đặc trưng cho sự kết hợp hoàn giữa đại số và hình học Những tính chất của vector lại mang đến lời giải thật sáng sủa và đe ̣p mắt Nhưng số lượng các bài toán của phương pháp này không nhiều Trước đến với những ví dụ cách chứng minh bất đẳng thức cách đưa vector và tích vơ hướng, ta cần ghi nhớ số đẳng thức sau:

_

 

       

2

2

A

* ,

* , , ,

* *

*

A B B B A B A

a x y a x y

A x y B x y AB x x y y

a b a b a b a b a b a b

   

    

  

  

 

Ta có phương pháp chung cho tốn bất đẳng thức sử dụng đưa vector tích vơ hướng là: Khi gặp tốn bất đẳng thức nói riêng tập đại số nói chung mà biểu thức dấu căn( A, B) biểu diễn dạng tổng hai bình phương ta cố gắng tìm hệ trục toạ độ Đềcác vector có độ dài A, B Sau nghiệm lại tổng vector khơng có vector tổng vector cịn lại sử dụng BĐT độ dài cạnh tam giác BĐt độ dài đường gấp khúc để đến kết tốn Cịn với tốn hình học ta đưa chúng véctơ đơn vị

Để làm sáng tỏ điều vừa nêu sau có số ví dụ tập:

Ví du ̣

CMR mọi tam giác ta có :

2 cos cos

cosABC

Lờ i giải :

(42)

 

     

 

2 cos cos

cos

0 cos cos

cos

0 , cos , cos , cos

0

1 3

2

1

 

 

 

 

 

 

  

C B

A

C B

A

e e e

e e

e e e e

đpcm

Ví dụ 3: Cho x,y,z ba số tuỳ ý, Chứng minh rằng:

   

3 3

, , , , ,

2 2 2

3

;

2

y z x

a x y b y z c z x

a b c x y z x y z

     

          

     

 

        

 

Lời giải:

Trong mặt phẳng toạ độ, xét điểm

3 3

, ; 0, ; ,

2 2 2

y y z

A x  z B yz C  

 

   

Khi ta có:

 

2

2

2

2

2

2

3

2

3

2

3

2 2

y

AB x y x xy y

z

AC x z x xz z

y z

BC y z y yz z

 

 

        

   

 

 

        

   

 

 

         

   

Khi (1)  AB ACBC

Ví dụ 4: Chứng minh xta có:

2    

2x 2x 1 2x  1 x 1 2x  1 x 1 (1) Lời giải:

Ta có: (1)  

2 2 2

2

2 3

1

(43)

Trên mặt phẳng toạ độ xét  0,1 ; 3, ; 1,

2 2

A B   C 

   và T(x,x)

(1)TA+TB+TC >3

Dễ thấy AB=BC=CA OA=OB=OC tam giác ABC có tâm gốc toạ độ O

Bổ đề: Nếu tam giác ABC TA+TB+TC > OA+OB+OC, O tâm tam giác

Từ bổ đề ta có TA+TB+TC>OA+OB+OC Mà OA=OB=OC=1 TA TB TC  3(đúng) dấu xảy x=0

Ví dụ 5: Cho x,y,z số dương x+y+z1 Chứng minh rằng: 2

2 2

1 1

82

x y z

x y z

     

Lời giải:

Với u v, ta có (1)

( )

Đặt ,

áp dụng (1) ta có :

Vậy

Ta có :

Vậy P 82

(44)

2

2 2 2

xx  xx 

2 Với a,b hai số thực tuỳ ý ta có:

2

2 10 13 13 15 13

2 4

3 9 13

ab a a

(45)

2.4. Phương pháp quy nạp

Các nhà tốn học thường nói đùa “khi ta khơng thể giải tốn cụ thể lại giải tốn tổng qt khó khăn hơn” Đó có thực câu nói đùa, kinh nghiệm q báu trong tốn học? Khi tìm hiểu qua phương pháp quy nạp chứng minh bất đẳng thức thấy điều

Để phương pháp hiểu làm nhuần nhuyễn ta cần có kiến thức thật tốt phần này: “Chứng minh bất đẳng thức (*) với  n p(với (*) phụ thuộc vào số tự nhiên n, p số pN*) ta tiến hành bước sau:

Bước 1: Kiểm tra (*) với np

Bước 2: Giả sử (*) với * ,

n k p kN

Bước 3: Ta chứng minh (*) với n k

Bước 4: Kết luận bất đẳng thức (*) với np

Ví dụ 1: CMR:2n2n1với số nguyên dương n3 Lời giải :

Với n=3 2382.317hay bđt cho với n=3

Gỉa sử bđt với số ngun dương n=k ,có nghĩa : 2k2k12k.2(2k1)2

2k1 4k22k2k22(k1)1

Vậy bđt cho với n=k+1,do ta có nn nZ

,

2

n3

Ví dụ 2: CMR:2n n n

b a b

a n) ( )

(

1   

(1) với a+b>0,ab,n1

Lời giải:

Với n=2,(1)có dạng:2(a2 2 ) ( ) a b

b  

 (2)

Vì abta có bđt (a-b)20, cộng 2vế bđt với (a+b)2

ta có (2)

Gỉa sử (1) với số n=k , 2k1(akbk)(ab)k(3)

Để CM (1) cho n=k+1,ta nhân 2vế (3)với a+b a+b>0 ta nhận bđt :

2k1( kk)(  )(  )k1(4)

(46)

Như để chứng minh (1) với n=k+1 ta cần chứng minh

2k(ak1bk1)2k1(akbk)(ab)(5)

Sau biến đổi đơn giản vế ta bđt tương đương :ak1bk1 akbbka

từ ta suy ra:(akbk)(ab)0 (6) Xét trường hợp :

TH1:nếu a>b,và điều kiện cho a>-b,suy a>b,vì ak k

b

 Do bđt (6)

TH2:nếu a<b,lý luận tương tự phần tacó k k

b

a  ,trong trường hợp

(47)

2.5. Phương pháp phản chứng

Nói đến chứng minh bất đẳng thức ta không nhắc đến phương pháp phản chứng Phương pháp dùng đa dạng, đủ loại tốn phổ thơng chũng ta học Điều quan trọng phương pháp này giúp ln có lời giải sáng sủa đường chứng minh dễ dàng

Ví dụ 1: Cho a,b số dương ( )

b a

ab

<1.Chứng minh đồng thời xảy a<1 b<1

Lời giải : Giả sử đồng thời xảy a<1 b<1 a2 1,b2 1

Ta có: (1-a2 2 2 2

) ( )

( )

)( b    abab    abab

1 ) ( ) ( ) (

2 ) (

1 2 2 2 

   

 

 

   

b a

ab b

a ab ab

b a ab ab

BĐT cuối mâu thuẫn với giả thiết toán (1 )2 1

 

b a

ab

Vậy ta có đpcm

Ví dụ 2: Cho a,b,c,dRvà a+b=2cd CMR 2bđt sau

đúng c2a,d2 b

Lời giải :

Gỉa sử bđt sai ,có nghĩa ta :

c2 2 2

0 ,

0

,d b c a d b c

a       

 -a-b+d20

0 ) (

)

( 2

2

2           

c d a b c d cd c d

Kết mâu thuẫn ,vậy 2bđt cho phải Ví dụ 3: Gỉa sử x,y,z độ dài cạnh tam giác 1 1 1 3

z y

x tìm giá

trị nhỏ

P= xyzyzxxzy

(48)

Xét bđt : 2 2 2 2 2 2 2

) (

10

1

c b a a c c b b

a        

Cho a+b+c= 10 ,suy 2 2 2 2 2 2 1

   

b b c c a

a

Từ bđt ta suy bđt phản chứng

Nếu số dương a,b,c thỏa mãn điều kiện : 21 2 2 2 2 2 1

   

b b c c a

a

ta ln có a+b+c 10thật bđt tương đương với

Nếu a1 ,b1,c1>0 thỏa mãn 1 1

1 1

  

c b

a

a1 b1c1  b1c1a1  a1c1 b1 2 Do 1 1 3

z y

x

5

       

y z y z x x z y

(49)

2.6 Phương pháp dùng tam thức bậc hai

Trong phương pháp sử dụng định lí dấu tam thức bậc hai để chứng minh bất đẳng thức

Muốn chứng minh bất đẳng thức cần đưa bất đẳng thức cần chứng minh dạng A(*)

Khi ta xem vế trái (*) tam thức bậc hai biến nào sử dụng định lí thuận định lí đảo tam thức bậc hai để chứng minh (*)

Dạng 1 : Sử dụng định lí thuận dấu tam thức bậc hai *Định lí dấu tam thức bậc 2:

Cho tam thức bậc :  

:ax 0( 0)

f xbx c a

+ Nếu  0thì af(x)>0 với x + Nếu  0thì af (x) 0với x Dấu đẳng thức xảy

a b x

2

 

+Nếu 0 lập bảng xét dấu

Ví dụ 1: Cho a,b,c cạnh tam giác x,y,z ba số thỏa mãn điều kiện ax+by+cz=0.Chứng minh xyyzzx0(1)

Lời giải:

Từ ax+by+cz=0

c by ax z 

 Vậy:

(1)      (xy)

c by ax xy zx yz xy

0 )

(

2     

ax xy a b c by (2)

Nếu y=0 (2) ax2 0(2) (1)

(50)

(2) ( )

  

       

b

y x c b a y

x a

Quan niệm vế trái (3) tam thức bậc hai

y x

có hệ số

2

     

y x

là a>0 (abc)2  4aba2  b2  c2  2ab2ac 2bc

Từ |b-c|<a=> 2

2bc c a

b    , tương tự a2 2bcc2 b2

2

2bc b c

a   

Vậy a2 bc2 2ab2bc2ca=>a2bc22ab2bc2ca0 nên vế trái (3) lớn => (3) => (1) chứng minh Dấu "=" xảy x = y = z =

Ví dụ 2: Cho a3 36 abc=1.Chứng minh rằng:

2

3

a

b c ab bc ca

    

Lời giải:

Từ abc=1 =>

a

bca3 36 nên chắn a>0.Ta có:

) (

) (

2

c b a bc bc c

b

a      

0 3

) ( ) (

2

2     

b c a b c bc a (1)

Xét tam thức bậc hai

3

) (

2

2 a

bc ax x

x

f    

Ta có hệ số x2 1>0 12 36 0

2

2   a

bc a

(51)

Theo định lí thuận dấu tam thức bậc hai f(x)>0 với x => f(b+c) = (b+c)2 – a(b+c)-3bc+

3

2

a

đúng =>đpcm

Dạng 2: Sử dụng định lí đảo dấu tam thức bậc hai : *Định lí đảo dấu cho tam thức bậc 2:

Cho:f(x): ax2+bx+c = (a 0)

Nếu tồn a R cho af(x)<p f(x) có nghiệm pb x1 < a < x2

Ví dụ Cho (a+c)(a+b+c)<0 Chứng minh: (b-c)2>4a(a+b+c)

Lời giải:

Nếu a=0 từ giả thiết ta có c(b+c)<0 (1)

Bất đẳng thức phải chứng minh có dạng (b-c)2>0 (2) Từ (1) suy b cvậy (2) => đpcm

Nếu a xét tam thức bậc hai sau : f(x) = ax2+(b-c)x+a+b+c

Từ f(0)=a+b+c ; f(-1)=2(a+c) >từ gải thiết ta có f(0)f(-1)<0.Theo định lí đảo dấu tam thức bậc hai suy phương trình f(x)=0 có hai nghiệm phân biệt Hay (b-c)2 – 4(a+b+c)>0(b-c)2 > 4(a+b+c) đpcm

(52)

2.7 Sử dụng số bất đẳng thức phụ

Trong phương pháp sử dụng sô bất đẳng thức phụ để chứng minh bất dẳng thức khác

Để vận dụng bất đẳng thức ta cần đưa bất đẳng thức cần chứng minh dạng chúng

Sau số bất dẳng thức phụ cách chứng minh chúng số ví dụ

Bất đẳng thức I : Cho a,b>0 :

2

 

a b b a

( I )

Chứng minh: Áp dụng BĐT Cauchy cho số dương a,b ta

2

2  

 

a b b a a

b b a

(đpcm)

Ví dụ :Cho a,b,c,d số dương abcd =

Chứng minh rằng: a2 b2 c2 d2 abacadbcdbcd 10 Chứng minh:

Theo BĐT Cauchy ta có :a2b2 2ab(1) c2d2  2cd(2) (vì a,b,c,d > 0)

Dễ thấy ab,cd > abcd = nên

cd ab

2

   

cd

cd cd

ab (3)(Áp dụng BĐT (I))

(53)

2ab2cd  4(6)

Cộng theo vế (1),(2),(3),(4),(5),(6) ta đpcm Bất đẳng thức II : Cho a,b,c>0 :

  1 19()

  

   

c b a c b

a

Chứng minh:

Để chứng minh BĐT (II) ta sử dụng BĐT Cauchy cho số dương

c b a

1 , ,

và a,b,c sau:

Ta có:

c b a abc abc

c b a

abc c

b a

   

  

  

3

, 1

3

3

Khi ta có:

c b a c b

a    

9

1

Đây BĐT cần chứng minh

Vấn đề đặt giải tốn chưa có dạng ta phải biến đổi

Ta xét ví dụ sau:

VD1: Cho a,b,c 0 chứng minh:

3

       

a c

b c

b a c b a

c a

(2) Chứng minh:

Để giải toán dạng ta xây dưng cách giải sau: Ta biến đổi để xuất dạng (II)

Trước hết ta tìm số x,y cho:

M c b y a c b a x c

a3 ) (  ) ( 3 ) (  ) 

(

c y yb

a c xb a

x) 3 3 (1 )

1

(       

sử dụng phưong pháp đồng thức ta có :

  

   

 

  

c b a M y

x y

x

3 ,

2 ,

3

(54)

Bây ta tìm m,n cho: ) 3 2 3 ( ) (

4bm can abc

Tương tự ta tìm m=6,n=2 Đến ta chứng minh (2) sau Ta có (2) tương đương với

16 3                 a c b c b a c b a c a tức 16 1 ) 3 (               a c c b b a c b a

VP tương đương với

       

  1 1  16

              a c a c c b b a a c a c c b b a (đúng) =>đpcm

Bây ta dùng BĐT (II) để chứng minh số BĐT khác: Ví dụ 2: Cho a,b,c >0;CMR

c b a a c c b b

a       

3 2 Chứng minh:

Ta vận dụng BĐT(II)cho3 số dương là2a+b,2b+c,2c+a ta có

c b a a c c b b a c b a a c c b b a a c c b b a a c c b b a                              2 ) ( 2 2 2 2

(55)

Ta có : a3+b3  a2b + ab2 (a+b)(a2 – ab + b2)  ab(a+b)

 (a+b)(a2– ab +b2 - ab)   (a-b)2(a+b)  0.(Đúng)

Ví dụ : Cho a,b,c>o.CMR:

2

2

3 3 3

c b a ca

a c bc

c b ab

b

a        

Lời giải:

Ta sử dụng BĐT (III) sau :

Ta có a3+b3  a2b + ab2  a3+b3  ab(a+b) 

ab b a

2

3 

2 b a

; Tương tự, ta có:

2

, 2

3 3

3

a c ca

a c c b bc

c

b

  

 

Cộng theo vế BĐT lại với ta BĐT cần chứng minh Bất đẳng thức IV Cho a, b số dương ta ln có:

1

b a b

a    (IV)

Chứng minh

Cách 1: (Dùng phương pháp biến đổi tương đương) Ta có BĐT

b a b

a    1

b a ab

b a

 

(a+b)2  4ab (a-b)2  BĐT với a, b nên BĐT

(IV) với a,b dương

Cách 2: (Dùng BĐT Cauchy cho hai số dương ) Áp dụng BĐT Cauchy cho hai số dương là:

b a

1 ,

ta có:

ab ab

b a

2

2 1

 

 , lại áp dụng BĐT Caychy cho hai số dương a vvà

b, ta có: a+b

b a ab ab

  

 2 Từ hai BĐT ta BĐT (IV)

(56)

2 ) ( 4 b a ab b

a    

Lời giải:

Ta vận dụng BĐT (VI) để chứng minh BĐT sau: Ta áp dụng BĐT (VI) cho hai số 4a2+4b2 8ab Ta có:

2 2 2 2 ) ( 4 4 4 b a ab b a ab b a ab b

a          

Đây BĐT cần chứng minh

Bất đẳng thức V: Cho a, b số dương, ta ln có:

) ( b a

ab  (V)

Chứng minh

Cách 1:(Dùng phương pháp biến đổi tương đương)

Ta có BĐT (4) (ab)2 4ab(ab)2 0 Đây BĐT

Cách 2: (Dùng BĐT Cauchy)

Ta áp dụng BĐT Cauchy cho số dương a b, ta cã: ab

2 ) ( ) ( b a ab b a     

Ví dụ : Cho a, b, c số dương CMR:

2 ) ( ) )( ( ) )( ( ) )( ( c b a a c a b c a c b b c b

a          

Chứng minh

Ta vận dụng BĐT (V) cho số dương 2a+b 2c+b ta có:

2 2 ) ( ) )( ( ) 2 ( ) )( ( ) 2 ( ) )( ( c b a b c b a c b a b c b a b c b a b c b a                  

(57)

2

) (

1 )

2 )( (

1

, ) (

1 )

2 )( (

1

c b a a c a b

c b a c a c b

    

    

(58)

2.8 Sử dụng định lí Viét

Đây phương pháp chứng minh bất đẳng thức hay và mạnh giúp người chứng minh có thể sử dụng cách có hiệu quả tiết kiệm thật nhiều thời gian cho việc chứng minh Sau số ví dụ giúp thấy sức mạnh phương pháp

Ví dụ 1: Cho số thực x,y,z khác thoả mãn điều kiện x + y +z =xyz,

x = yz Chứng minh rằng: x2 3

Lời giải: Dễ thấy

3

y+z =

{ x x

yz x

 

Vậy y,z nghniệm phương trình:

( )

u  x x ux  (1) Ta có: 2

((1 ) 4)

x x

    (2) Bởi (1) có nghiệm  0, x0, nên từ (2) suy

 22  22 1x    4 x 4 1 x2 2hoặc1 x2 -2

Dễ thấy 1-

x 2vơ nghiệm, ta có 1 x2-2

x 3(đpcm)

Ví dụ 2: Các số thực x,y,z thoả mãn điều kiện x + y + z =5 yz + xz +xy =8 Chứng minh rằng:

1 x,y,z

3

Lời giải:

Từ điều kiện tốn ta có:

8 ( )

{y z x

yz x x

     

(59)

 2 2

5 4( 8)

7

3

x x x

x

 

     

  

Vai trò x,y,z nên ta có

1y,z

3

 (đpcm)

Ví dụ 3: Giả sửx1,x2 nghiệm phương trình

x + kx + a = (a 0) Tìm tất giá trị k để có bất đẳng thức:

3

1

2

52

x x

x x

   

 

   

    ( a,kR)

Lời giải:

Ta xét a trường hợp:

* Nếu a<0 thì=k2 4a0với k Khi đố phương trình cho ln có

hai nghiệm khác khác dấu Điều dẫn đến bất đẳng thức cho đũng với kR

* Nếu a>0

Ta có x3  y3 (xy x)( 2xyy2) Áp dụng công thức ta có

3

1

2

x x

x x

        

    = 12 21

x x x x

 

 

 

2

2

3 x x

x x

  

    

  

 

nhưng 2

x x x x

 

 

 =

2 2

1

x x x x

= 

2

1 2

1 2

x x x x x x

 

=

2 k

a

 

 

 ( Theo định lí Viét)

Do

3

1

2

x x

x x

   

   

    =

2

2 k

a

 

 

 

2

2

k a

  

    

  

 

đặt

k t

(60)

(t-2)(t22 3)52

(t-6)(t2+ 9)0 Ta thấy

t + 9>0 với t,do t-60, hay

k

a -60

k  6a(do a>0)

- 6a k 6a

Vậy với

0

{a

k R  

Hoặc

0

- 6a 6

{a

k a

 

Thì

3

1

2

52

x x

x x

   

 

   

   

(61)

2.9 Bài tập :

Bài 1: Cho ABCbất kỳ CMR :

2 tan tan tan cot

cot cot

2 2

2 2

C B A

c b a C

B A

c b

a

   

 

 

 

Bài 2: Cho ABCbất kỳ CMR : R r S

3

 

Bài 3: Cho ABCnhọn CMR :

2

cos

cos

cos ABC

Bài 4: Cho x,y,z >0 Chứng minh rằng:

 

2 2 2

3

xxyyxxzzyyzzx y z

Bài 5: Tìm GTNN hàm sốyx22px2p2  x22qx2q2 (p#q)

Bài 6: Chứng minh rằng:

  2 2

2 2

mnpqmpnqm n p q, , , R Bài 7: Chứng minh bất đẳng thức sau:

n

1

1

1  > n (n=2,3,) (1)

Bài 8: Chứng minh (xy)2 2x 55y24y 56,x,y.(*) Bài 9: (đề thi HSG tỉnh)Cho ai>0; i = 1, 2, 3, CMR:

1

2

4

1

3

4

4

3

1 

   

      

a a

a a a a

a a a a

a a a a

a a

.(1)

Bài 10: Cho a,b,c,d số dương.chứng minh:

     

a b

d a d

c d

c b c b

a

(62)

Bài 11: Cho a,b,c >0 Chứng minh :

c b a ab

c ac

b bc

a     

Bài 12: Cho a,b,c 0 chứng minh:

6

3 

      

a c

b c

b a c b a

c a

Bài 13: Giả sửx1,x2 nghiệm phương trình

x + 2kx + =0 Tìm tất cả giá trị k cho có bất đẳng thức:

2

1

2

3

x x

x x

   

 

   

(63)

Chương :

Áp du ̣ng vào mô ̣t sớ vấn đề khác

“Có học phải có hành”

Sau xem xét bất đẳng thức phương pháp chứng minh ta phải biết vận dụng kết vào vấn đề khác

Trong chương trước ta có ví dụ bất đẳng thức lượng giác mà dấu thường xảy trường hợp đặc biệt : tam giác đều, cân hay vng …Vì lại phát sinh dạng : định tính tam giác dựa vào điều kiện cho trước

Mặt khác với kết chương trước ta dẫn đến dạng tốn tìm cực trị lượng giác nhờ bất đẳng thức Dạng hay : kết “giấu” đi, bắt buộc người làm phải tự “mò mẫm” tìm đáp án cho riêng Cơng việc thật thú vị ! Và tất nhiên muốn giải tốt vấn đề ta cần có “vốn” bất đẳng thức “kha khá”

Bây kiểm tra hiệu bất đẳng thức lượng giác chương : “Áp dụng vào số vấn đề khác”

Mục lục :

3.1 Định tính tam giác 3.1.1 Tam giác 3.1.2 Tam giác cân 3.1.3 Tam giác vuông

(64)

3.1 Định tính tam giác : 3.1.1 Tam giá c đều :

Tam giá c đều có thể nói là tam giác đe ̣p nhất các hình mà ta biết Ở nó ta có được sự đồng nhất giữa các tính chất của các đường cao, đường trung tuyến, đường phân giác, tâm ngoại tiếp, tâm nội tiếp, tâm bàng tiếp tam giá c … Và các dữ kiê ̣n đó lại cũng trùng hợp với điều kiê ̣n xảy dấu bằng ở các bất đẳng thức lượng giác đối xứng tam giác Do đó sau giải được các bất đẳng thức lượng giác thì ta cần phải nghĩ đến viê ̣c vận dụng nó trở thành một phương pháp nhận dạng tam giác đều

Ví du ̣

CMR ABC đề u thỏa : ma mb mc R

2

  

Lờ i giải :

Theo BCS ta có :

   

   

m m mRA B C

c b a m

m m

m m m m

m m

c b a

c b a

c b a c

b a

2

2 2

2 2

2 2

sin sin

sin

4

 

 

 

  

  

  

 

mà :

4 sin

sin

sin2 A 2B 2C

 

R m

m m

R R

m m m

c b a

c b a

2

4 81

9 2

2

   

   

 

Đẳng thứ c xảy và chỉ ABC đều đpcm

Ví du ̣

(65)

Ta có :   cos sin cos cos sin 2 cos sin sin sin sin

4 C A B

B A C C R B A B A R C R B A R c b a c ab            sin cos cos 2 cos cos cos cos cos cos sin sin cos cos sin sin 2                                        B A B A B A B A B A B A B A B A B A B A B A B A B A đpcm

Ví du ̣

CMR ABC nó thỏa : 2hahbhc  abc

Lờ i giải :

Điều kiện đề bài tương đương với :

  cot cot cot cot cot cot 3                       A C C B B A c r b r a r c b a c r b r a r p

Mặt khác ta có :

                       tan tan cot cot cot cot

1 A B

B A

B A

(66)

                  tan tan cot cot tan tan cot cot A C A C C B C B tan tan tan tan tan tan 2 tan tan tan 2 cot cot cot cot cot cot                              C B A C B A C B A A C C B B A đpcm

Ví du ̣

CMR nế u thỏa

2 3Rr

S thì ABC đề u

Lờ i giải : Ta có :

Rr R r C B A R r C B A R C B A R C B A C B A R C B A R S 3 3 cos cos cos cos cos cos sin sin sin cos cos cos sin sin sin sin sin sin

2 2

      đpcm

Ví du ̣

CMR ABC đề u nó thỏa mambmcpS

(67)

   

p a

p m bc a p p bc a c b bc bc a c b A bc a c b A bc a c b A

a  

                   4 cos 2 cos 2 cos 2 2 2 2 2 2

Tương tự :

 

 

p ap bp cpS p p m m m c p p m b p p m c b a c b               đpcm

3.1.2 Tam giá c cân :

Sau tam giá c đều thì tam giác cân cũng đe ̣p không kém Và ở thì chú ng ta sẽ xét những bất đẳng thức có dấu bằng xảy hai biến bằng nhau và khác biến thứ ba Ví dụ

3 ;    

B C

A Vì thế nó khó trường hợp xác ̣nh tam giác đều

Ví du ̣

CMR ABCcân nó thỏa điều kiê ̣n

2 tan tan

tan2 ABAB và nhọn

Lờ i giải :

Ta có :        

C B A C B A B A B A B A B A B A cos cos sin cos cos sin cos cos sin tan tan           

vì    

2 sin cos cos cos

cos AB   ABC  CC

  tan tan tan tan 2 cot 2 sin 2 cos sin sin sin cos cos sin 2 B A B A B A C C C C C C C B A C            

Từ giả thiết :

2 2 2 tan tan 2 tan tan tan          

B A B A B

A

(68)

  B A B A B A        tan tan tan tan đpcm

Ví du ̣

CMR ABC cân thỏa

2 cosA

bc ha

Lờ i giải :

Trong mọi tam giác ta có :

2 cos A c b bc l

ha a

 

mà bc

bc bc c b bc bc c

b  

  

 2

cos cos cos A bc h A bc A c b bc a     

Đẳng thứ c xảy ABC cân đpcm

Ví du ̣

CMR nế u thỏa

2 sin 4R B r

ra thì ABC cân

Lờ i giải : Ta có :

(69)

Ví du ̣

CMR nế u  2

1

b a

S   thì ABC cân

Lờ i giải :

Ta có : abab ab  ababsinCS

2

1

1

2 2

2

     a2 b2 S

4

ABC

 cân nếu thỏ a điều kiê ̣n đề bài

Ví du ̣

CMR ABC cân thỏa

4 cos cos

cos

2 ABC

Lờ i giải : Ta có :

4 9 sin

cos 2 sin

4 cos

cos 2 sin cos sin 2 sin

2 cos cos 2 sin 2 cos cos

cos

2

2

2

2

   

   

 

 

   

   

  

     

 

 

    

    

C B C

B A

C B C

B A

C B A A

C B C B A

C B

A

(70)

3.1.3. Tam giá c vuông :

Cuố i cù ng ta xét đến tam giác vuông, đại diê ̣n khó tính nhất của tam giác đối với bất đẳng thức lượng giác Dường nhận diê ̣n tam giác vuông, phương pháp biến đổi tương đương các đẳng thức là được dùng cả Và ta hiế m gặp bài toán nhận diê ̣n tam giác vuông mà cần dùng đến bất đẳng thức lượng giác

Ví du ̣

CMR ABC vuông thỏa 3cosB6sinC4sinB8cosC15 Lờ i giải :

Theo Bunhiacopxki ta có :

  

  

   

 

 

 

 

10 cos

sin cos

8 sin

5 sin

cos sin

4 cos

2

2

2

2

C C

C C

B B

B B

3cosB4sinB6sinC8cosC15 Đẳng thứ c xảy và chỉ :

2 cot

tan

4 cot

3 tan

8 cos

sin

4 sin

cos 10

cos sin

5 sin cos

3 

   

 

    

  

     

  

  

 

 

C B C B

C B C

C

B B

C C

B B

(71)

3.2 Vận dụng bất đẳng thức vào giải toán

Vận dụng bất đẳng thức vào giải toán phương pháp tối ưu để giải tốn hóc búa Đơi tóa dễ không biết vận dụng lúc gây khó khăn Vì vận dụng bất đẳng thức vào giải toán phương pháp chũng ta cần biết tới Để thấy điều ta có số ví dụ sau:

Ví dụ 1: Cho a, b, c số đo độ dài cạnh tam giác thoả mãn điều kiện:

(a+b)(b+c)(c+a) = 8abc

Chứng minh a, b, c cạnh tam giác

Giải:

*Vì a, b, c cạnh tam giác nên ta ln có:

ac c

a

bc c

b

ab b

a

2 2

 

 

 

*Vì a+b>0, b+c>0, c+a>0 số vế phải không âm nên ta viết:

8

) )( )(

(ab bc caa2b2c2  abc

Dấu đẳng thức xảy khi: a=b=c Từ ta có điều phải chứng minh

Ví dụ 2:

Tìm phần ngun số thực sau:

Ax2  4x2  16x2 8x3 với x số nguyên dương Giải:

Với x>0, ta có:

(4x+1)2<16x2+8x+3<(4x+2)2 Suy tiếp:

) 2 ( 16

)

( x  x2  x2 x  x

(72)

2

8 16

1 2

   

    

 

x A x

x x

x x

x x

Theo định nghĩa phần nguyên số A ta có [A]=x+1 Ví dụ 3:

Tìm tất nghiệm nguyên dương phương trình:

1 1

  

z y

x (1)

Giải:

Khơng tính tổng quát, ta giả sử xyz

Sau tìm ba số nguyên dương thoả mãn (1) ta hoán vị ba số nghiệm khác

*Theo giả thiết, theo điều giả sử ta có:

z z y x

3 1

2   

2z  mà z nguyên dương, z=1 *Thay z=1 vào (1), ta được:

1 1 

y x

Theo giả sử xy nên có:

y y x

2 1 1  

 y2, mặt khác y nguyên dương, y=2 y=1 Giá trị y=1 khơng thích hợp, cịn với y=2 ta cso x=2

Thử lại 2

1

1  

thoả mãn (1)

(73)

Giải:

*Với x=0, y=0 ta nghiệm thoả mãn phương trình (1)

*Ta tìm nghiệm nguyên dương thoả mãn (1) x<0, y<0 phương trình (1) khơng có nghĩa

Bình phương hai vế ta được:

y x x 

xy2 x số nguyên dương

Đặt xk (k nguyên dương)

2 ) (k y

k  

Nhưng 2

) )

(   

k k k

k

Suy ra: k2  y2 (k1)2 kyk1 k>0, y>0

Nhưng hai số nguyên dương liên tiếp không tồn số nguyên dqương khơng có cặp số ngun dương thảo mãn (1)

(74)

3.3 Bài tâ ̣p :

Bài 1: CMR ABC đề u nếu nó thỏa một các đẳng thức sau :

a

4 cos cos cos

cos cos

cosA BB CC A

b sin2Asin2Bsin2CsinAsinBsinC

c A B C

C B

A 2tan tan tan

1

3

sin

sin

sin

1    

d

2 tan tan tan cot

cot cot

2 2

2 2

C B A

c b a C

B A

c b a

    

 

 

 

Bài 2: Giải phương trình:

5 6

4

3  

  

  x x

Bài 3:Chứng minh vơí số tự nhiên ta ln có:

nn1  4n2

Bài 4: Tìm năm số thực dương cho số bình phương tổng

bốn số lại

Bài 5: Cho tam giác ABC Ở miền tam giác có điểm M Các đường

thẳng AM, BM CM cắt cạnh tam giác điểm A1,

B1, C1 thoả mãn điều kiện:

Chứng minh M trọng tâm tam giác ABC

Bài 6: Cho tứ giác ABCD có diện tích S, hai đường chéo AC BD cắt

nhau O

Gọi s1, s2 diện tích tam giác AOB tam giác COD

Chứng minh AB//CD s1  s2  S

Bài 7: Cho tam giác ABC Một đường thẳng song song với cạnh BC cắt AB

tại D, cắt AC E Chứng minh với điểm P cạnh BC ta ln có diện tích tam giác PDE khơng lớn ¼ diện tích tam giác ABC Đường thẳng DE vị trí tam giác PDE đạt giá trị lớn nhất?

Bài 8: Cho hình bình hành ABCD P điểm miền hình

(75)

Bài 9: Giải phương trình: x x x

5

3   (1) tập số nguyên Bài 10: Giải phương trình: 4 6  10 27

x x

x x

Bài 11: Giải hệ phương trình với x, y, z số dương:

    

 

 

 

1 1

2 2

x z

z y

y x

) (

) (

) (

Bài 12: Giải phương trình: a) x24x16 2x200

b) x4 x2 2 2x20 Bài 13: Giải phương trình:

        

 

 

 

x z z

z y y

y x x

2

2

2

2

2

2

2

) (

) (

) (

Bài 14: Giải phương trình:

1

1 4

4      

x x

(76)

Chương :

̣t số chuyên đề bài viết hay, thú vi ̣ liên quan đến bất đẳng thức

Đú ng tên go ̣i của mình, chương này sẽ bao gồm các bài viết chuyên đề về bất đẳng thức Tác giả của chúng đều là các giáo viên, ho ̣c sinh giỏi toán mà tác giả đánh giá rất cao Nô ̣i dung của các bài viết chuyên đề đều dễ hiểu và ma ̣ch la ̣c Chúng ta có thể tham khảo nhiều kiến thức bổ ích từ chú ng Vì khuôn khổ chuyên đề nên chỉ tâ ̣p hợp được mô ̣t số bài viết thâ ̣t sự là hay và thú vi ̣ :

Mục lu ̣c :

4.1 Về bất đẳng thức có nhiều cách chứng minh

4.2 Ứng dụng đại số vào việc phát chứng minh bất đẳng thức tam giác

4.3 Thử trở cội nguồn mơn Lượng giác 4.4 Từ tốn quen thuộc bất đẳng thức 4.5 Những toán không thuộc dạng thông thường

(77)

4.1 Về bất đẳng thức có nhiều cách chứng minh

Bài viết giới thiệu phương pháp chứng minh bất đẳng thức Nội dung toán cần giải sau:

“Các số a,b,c,d thõa mãn đẳng thức 2 2 2 2

+ = 1, + = 1

a b c d Chứng minh

rằng ac bd- ≤ 1”

4.1.1 Phương pháp biến đổi tương đương

Ta có: ac bd- ≤ (1)

( )2

-ac bd ≤1

2 2 2 2

- 2 +

a c abcd b d ≤1

2( 2)2 2( 2)2

1 - - 2 + 1

-a d abcd b c ≤1

2 2 2 2 2 2

- - 2 +

-a a d abcd b b c ≤1

2 2 2 2

-a d - 2abcd b c- ≤0

( )2

+

ad bc ≥0 (2)

Dễ thấy (2) đúng, (1)

4.1.2 Phương pháp tổng hợp

Xuất phát từ bất đẳng thức cần chứng minh, ta có:

-ac bd ≤ 1

1

ac bd

ac bd

 

     

2

2 2

ac bd

ac bd

 

    

Từ để ý đến bất đẳng thức    

   

2

2

0

a c b d

a c b d

   

   

Đến ta có

   

   

   

   

2 2 2

2 2 2

2

0

2

0

2 2

2 2

1

a b c d ac bd

a c b d

a b c d bd ac

a c b d

ac bd ac bd

bd ac ac bd

ac bd

         

 

 

    

   

 

 

   

 

 

    

 

  

(78)

4.1.3 Phương pháp phản chứng

Giả sử acbd >1, nghĩa

1

ac bd

ac bd

 

  

> <

     

   

   

   

2 2

2 2

2

2

2

0

ac bd a b c d

ac bd a b c d

a c b d

a c b d

    

 

     

   

 

   

> <

< <

Điều khơng xảy Vậy ta có: ac bd-

4.1.4 Phương pháp dùng bất đẳng thức quen thuộc

Áp dụng bất đẳng thức x y-x + y ta có:

-ac bdac + bd = a c2 2 + b d2 2 (1)

Mặt khác, áp dụng bất đẳng thức Côsi cặp số 2 2

à c

a v , 2 2

à d

b v , ta có:

2 2 2 2

+ +

+

2 2

a c b d

2 2 2 2

+

a c b d

2 2 2 2

+ + +

2

a b c d

2 2 2 2

+

a c b d

1≥ 2 2 2 2

+

a c b d (2)

Từ (1) (2) suy ac bd- ≤ (dpcm)

4.1.5 Phương pháp đánh giá

Trong trường hợp này, từ điều kiện cho trước tốn, thơng qua việc biến đổi đồng , ta có:

 2 2 2 2 2 2

a b c d a c a d b c b d

      

       

   

2 2

2

2

ac abcd bd ad abcd bc

ac bd ad bc

     

(79)

     

     

2 2

2 2

ac bd ad bc ac bd ac bd ad bc ad bc

     

     

Nghĩa  2 1 acbdacbd 1

4.1.6 Phương pháp dùng tích vơ hướng hai vectơ

Đặt x=(a b;- )

y=(c d; )

(Điều xuất phát từ hai đẳng thức cho trước) Giả sử α góc xen hai vectơ x y,

Ta có x y=ac bd

x y. = ac bd

-Mặt khác x y. = x y c αosx y (vì ( 2 2)( 2 2)

= + + à cos

x y a b c d v α ≤1)

Nghĩa ac bd- = x y.x y = 1

4.1.7 Phương pháp lượng giác

Trên đường tròn lượng giác, ta lấy hai điểm A(a;b) , b(c;d) Ta kí hiệu

,

α β góc lập bán kính OA OB (O gốc tọa độ) với trục Ox (chiều dương) Khi đó:

a=cosα, b=sinα c=cosβ, d=sinβ

Dễ thấy ac bd- = os cos -sin sinc α β α β = osc (α β+ )≤1

Nghĩa ac bd-

Đôi điều kết luận: Bài tốn khơng xa lạ, cách giải toán

(80)

4.2 Ứng dụng của đa ̣i số vào viê ̣c phát hiê ̣n và chứng minh bất đẳng thứ c tam giác

1/ Chúng ta từ toán đại số sau: Với  

 

x

  

ta ln có:

x x 2x

< tg < < sinx < x

2 2 π .

Chứng minh: Ta chứng minh bất đẳng thức: sinx 2x

2

x x

tg

  Đặt f x( ) 1sinx

x

 hàm số xác định liên tục 0, 

 

 

 

Ta có: f '( )x xcos x- sin x2 x

 Đặtg x( )xcos x- sin x 0,

    

 

   

' sin

g x  x x g x nghịch biến đoạn 0, 

 

 

  nên g x g 0 =0

với 0,

x 

  Do f ' x 0với x 0,2

 

   suy f x  f 2

     

  hay

2 sinx x

 với 0,

2

x       Đặt h x  1tgx

x

 xác định liên tục 0, 

 

 

 

Ta có  

2 sin

'

2 os

x x

h x

x x c

  0,

2

x   

   nên hàm số h x  đồng biến,

đó  

2

x h xh  

  hay

2

x x

tg

 với 0,

x  

   Còn bất đẳng thức

2

x x

tg  sinxx dành cho bạn đọc tự chứng minh

Bây phần đáng ý:

Xét ΔABC: BC = a, BC = b, AC = b Gọi A, B, C độ lớn góc radian; r, R, p, S bán kính đường trịn nội tiếp, bán kính đường trịn ngoại tiếp, nửa chu vi diện tích tam giác; la, ha, ma, ra, tương ứng độ dài đường phân giác, đường cao, đường trung tuyến bán kính đường trịn bàng tiếp ứng với đỉnh A

(81)

Bài toán 1: Chứng minh tam giác ABC nhọn ta ln có:

2 2

os os os

4

p p

Ac x Bc B Cc C

R R

   

Nhận xét:

Từ định lí hàm số sin quen thuộc tam giác ta có:

sinA sinB sinB p R

   bài toán đại số ta dễ dàng đưa biến đổi sau

2

os os sin os

2

A

Ac A tg c A A Ac A

   , từ đưa đến lời giải sau Lời giải:

Ta có: 2

os os sin os

2

A

Ac A tg c A A Ac A

   

os sin p

Ac A A

R

 

 

và 2

os sin os

4

p p

Ac A A Ac A

R R

      Từ suy đpcm

Trong tam giác ta có nhận xét sau:

2 2 2

A B B C C A

tg tgtg tgtg tg  kết hợp với

2

x x

tg

 nên ta có2 2 2

2 2 2

A B B C C A A B B C C A

tg tg tg tg tg tg

           

4

A BB CC A (1) Mặt khác

2

x x

tg  nên ta dễ dàng có

1

2 2 2 2 2 2

A B B C C A A B B C C A

tg tg tg tg tg tg

      từ ta lại có

A BB C C A  (2) Từ (1) (2) ta có tốn

Bài toán 2: Chứng minh tam giác ABC nhọn ta ln có:

2

4 A B B C C A

   

Lưu ý: Khi dùng cách để sáng tạo toán đề tốn ABC

phải nhọn tốn đại số 0,

x   

   Lời giải toán tương

tự nhận xét Mặt khác, áp dụng bất đẳng thức

 2

3

a b c

ab bc ca     ta có  

2 2

3

A B C

A BB CC A    Từ ta có tốn “chặt” “đẹp” hơn:

2

4 A B B C C A     

Bây ta thử từ cơng thức la, ha, ma, để tìm công thức

Trong ABC ta có:2 sin sin sin

2

a a

A A

(82)

 1 1

A 2 2

2 os

a

b c b c

l bcc bc b c

   

     

 

1 1 1 1 1

2 sin sin sin

a b c

l l l a b c R A B C

 

          

 

1 1 1 1

a b c

l l l R A B C

 

       

 

Như có Bài toán

Bài toán 3: Chứng minh tam giác ABC nhọn ta ln có:

1 1 1 1

a b c

l l l R A B C

 

      

 

Mặt khác, ta lại có sin sin 

A

2 os sin

2 2

a

R B C

bc b c

A

l c

 

 

  

 

 

Áp dụng bài toán đại

số ta được:

  2 

2

a

B C R

R B C bc

A

A l

 

 

 

  

   

 

4

a

R B C bc R B C

B C l B C

 

 

  

4

a

bc R

R l

 

Hồn tồn tương tự ta có:

c

ab R

R l

 

b

ca R

R l

  Từ đây, cộng chuỗi bất đẳng thức ta được:

Bài toán 4: Chứng minh tam giác ABC nhọn ta ln có:

12

3

c a b

R ab bc ca

R

l l l

    

Trong tam giác ta có kết sinA hb hc

c b

  , sinB hc ha

a c

 

sin ha hb

C

b a

  , mà từ kết bài tốn đại số ta dễ dàng có 2sinAsinBsinC, mà sin A sinB sinCha 1

b c

 

     

 

1 1

b c

h h

c a a b

   

      

    , từ ta có Bài tốn

Bài tốn 5: Chứng minh tam giác ABC nhọn ta ln có:

1 1 1

4 ha hb hc

b c c a a b

     

         

     

(83)

Nhận xét:Liên hệ với

a

m tam giác ta có

2 2

2

a

b c a

m    , từ ta

suy 2 3 2 2 2 2  sin sin sin

a b c

mmmabcR ABC từ đưa đến

lời giải Lời giải:

Áp dụng bài toán đại số ta được: 2 2

4

sin

x

x x

   ta

có:

2

2 2

4

sin

A

A A

   ,

2

2

sin

B

B B

  

2

2

sin

C

C C

  

Cộng chuỗi bất đẳng thức ta được:

 2 2 2 2 2

2

sin sin sin

A B C A B C A B C

         , mà ta có:

 

2 2 2 2

3 sin sin sin

a b c

mmmR ABC  

2 2

2 2

2 sin sin sin ,

a b c

m m m

A B C

R

 

    từ

đây ta được:  2 2 2 2 2

2

4

3

a b c

m m m

A B C A B C

R

 

      (đpcm)

Bây ta thử sáng tạo bất đẳng thức liên quan tới ra, ta có cơng

thức tính ra

2

a

A

rptg , từ bài toán đại số

2

x x x

tg

  chắn ta dễ dàng tìm thấy

2

a

r

A A

p

  , tương tự ta có

2

a

r

B B

p

 

2

a

r

C C

p

  ,

cộng chuỗi bất

đẳng thức ta thu 2 

2

a b c A B C

r r r

A B C

p

   

    và ta thu

Bài toán

Bài toán 7: Chứng minh tam giác ABC nhọn ta ln có:

 

2

a b c A B C

r r r

A B C

p

   

   

Ta tìm hiểu tốn sau:

Bài toán 8: Chứng minh tam giác ABC nhọn ta ln có:

2R raA bB cC 2 R r

      

Nhận xét: Ta có kết quả:

2

a

A rptg ,

2

b

B rptg ,

2

c

C

rptg ,  

2

A rp a tg 

   

2

B C

p b tg p c tg

    dẫn đến

2

a

A r  r atg ,

2

b

B r  r btg ,

2

c

C r  r ctg

4

a b c

r   r r R r (các kết bạn đọc tự chứng minh), từ ta suy

4

2 2

A A A

R r rptgptgptg nhờ kết ta dễ dàng đánh giá tổng

aA bB cC  từ toán đại số nên ta dễ có lời giải sau

(84)

Ta có:

2

a

A rptg ,

2

b

B rptg ,

2

c

C

rptg ,      

2 2

A B C

rp a tg  p b tg  p c tg ,

từ dẫn đến

2

a

A r  r atg ,

2

b

B r  r btg ,

2

c

C

r  r ctg Mà ta lại có:

4

a b c

r   r r R r suy

2 2

A A A

R r  rptgptgptg Áp dụng toán đại

số ta được:

●4 3 2 

2 2

A A A

R r r ptg ptg ptg r aA bB cC

        

2R raA bB cC

    

●4 3 1 

2 2

A A A

R r  rptgptgptgraA bB cC 

 

4 2R r aA bB cC

    

Kết hợp điều ta có điều phải chứng minh

Sau toán hình thành từ cơng thức quen thuộc để bạn luyện tập:

Bài toán: Chứng minh tam giác ABC nhọn ta ln có: a/ 2p8R raA bB cC  2p2Rr

b/         

2

S

p a p b p b p c p c p a S

          

c/ 2  2  2 

2

abca pab p b c p c  abc d/ la 1 lb 1 lc 1

b c c a a b

     

         

     

2/Chúng ta xét hàm:  

in

x f x =

s x với  x  0,

Ta có f x  hàm số xác định liên tục 0,

 

'

2 s inx-xcosx

sin

f x

x

 Đặt g x s inx-xcosx, x0,, ta có g x' xsinx0 

 

g x đồng biến đoạn 0, g x g 0 0 f' x 0 nên hàm

 

f x đồng biến

Chú ý bất đẳng thức đại số: 1. Bất đẳng thức Cauchy:

Cho n số thực dương a a1, 2, ,an, ta ln có:

aa  a

(85)

2. Bất đẳng thức Cauchy-Schwarz:

Cho n số a a1, 2, ,anvà b b1, 2, ,bntrong bi 0,i1,n Ta ln có:

 2

2 2

1 2

1 2

n n

n n

a a a

a a a

b b b b b b

      

  

Dấu “=” xảy 2

n n

a

a a

b b b

   

3/ Chúng ta xét bất đẳng thức sau: sinx 2x

π

với  

 

x

  

(phần chứng

minh bất đẳng thức dành cho bạn đọc)

Theo định lí hàm số sin ta có sin

a A

R

 kết hợp với bất đẳng thức

ta

2

a A a R

R    A  , từ ta dễ dàng suy

12

cyc

a R

A 

4/ Bất đẳng thức: sin x π - x22 22

xπ + x với   x   (bất đẳng thức xem

như tập dành cho bạn đọc)

Bất đẳng thức tương đương sinx 22x2 2

x   x

3 2

2 sinx x x

x

  

 (1)

Trong tam giác ta có: sin sin sin 3

ABC (2) (bạn đọc tự chứng

minh).Từ (1) (2) ta thu

3 3

2 2 2

3

sin

2 cyc

A B C

A A B C

A B C

  

 

        

  

 

 

3 3

2 2 2 3

2

A B C

A B C

  

 

     

  

 

3 3

2 2 2

3

2

A B C

A B C

       

Mặt khác, áp dụng bất đẳng thức cho góc A, B, C ta thu

2 2 sinA A

A A

 

 

 ,

2 2 sinB B

B B

 

 

2 2 sinC C

C C

 

 

 , cộng bất đẳng thức ta

được: sinA sinB sinC 22 A22 22 B22 22 C22

A B C A B C

  

  

  

    

   , từ áp dụng định lí

hàm số sin sin

a A

R

 ta có

2 2 2 2 2 2

2 2

a b c

A B C

R R R

A B C A B C

  

  

  

    

   hay

2 2 2

cyc

a A

R

A A

 

 

(86)

4.3 Thử trở cội nguồn môn lượng giác

“Lượng giác học” có nguồn gốc từ Hình học Tuy nhiên phần lớn học sinh học mơn Lượng giác học (giải phương trình lượng giác, hàm số lượng giác …), lại thấy phận môn Đại số học, cơng cụ để giải tốn hình học (phần tam giác lượng) mà không thấy mối liên hệ hai chiều môn

Trong viết này, tơi hy vọng phần cho bạn cách nhìn “mới” : dùng hình học để giải toán lượng giác

Trước hết, ta lấy kết quen thuộc hình học sơ cấp : “Nếu G trọng tâm tam giác ABC M điểm tùy ý mặt phẳng chứa tam giác thì” :

 2 2  2 2

1

1

c b a MC

MB MA

MG       (Định lý Lép-nít)

Nếu MO tâm đường trịn ngoại tiếp ABC 2 2 3R MB MB

MA   

nên áp dụng định lý hàm số sin, ta suy :

A B C

R R

OG2 2 sin2 sin2 sin2

9

4  

sin sin sin   1

9

4 2 2

2

   

   

OG R A B C

Từ đẳng thức  1 , suy :

 2 sin

sin

sin2 AB 2C

Dấu đẳng thức xảy GO, tức ABC Như vậy, với kiến thức hình học lớp 10 ta phát chứng minh bất đẳng thức  2 Ngoài ra, hệ thức  1 cho ta “nguồn gốc hình học” bất đẳng thức  2 , điều mà người nghĩ đến Bằng cách tương

tự, ta tính khoảng cách O trực tâm H ABC Xét trường hợp ABC

 có góc nhọn Gọi E giao điểm AH với đường tròn ngoại tiếp

ABC

 Thế :

 O OH R HE HA

H

2

/   

Do : OH2  R2 AH.HE  *

với :

A R C

A C R C

A AB C AF

AH cos

sin cos sin sin cos

sin   

(87)

cos cos cos  3

1 2

   

  

R A B C

OH

Nếu

90

BAC chẳng hạn,  3 hiển nhiên Giả sử ABC có góc A tù Khi H/ OR2 OH2 HA.HE AH 2RcosA nên ta suy  3

Từ công thức 3 , ta suy :

 4 cos cos

cosA B C

(Dấu đẳng thức xảy ABCđều)

Cũng bất đẳng thức  2 , bất đẳng thức  4 phát chứng minh với kiến thức lớp 10 có “nguồn gốc hình học” đẹp Cần nhớ rằng, “xưa nay” chưa nói đến việc phát hiện, riêng việc chứng minh bất đẳng thức đó, người ta thường phải dùng cơng thức lượng giác (chương trình lượng giác lớp 11) định lý dấu tam thức bậc hai

Có  1  3 , ta tiếp tục tiến tới Ta thử sử dụng “đường thẳng Ơle”

Nếu O, G, H tâm đường tròn ngoại tiếp, trọng tâm trực tâm ABC O, G, H thẳng hàng : OG OH

3

 Từ 2

9

OH

OG

Từ   1 ta có :

A B C 1 8cosAcosBcosC

4 sin

sin sin

4

9    

hay sin2 Asin2 Bsin2C22cosAcosBcosC

Thay 2

sin 2 cos

1 vào đẳng thức cuối cùng, ta kết quen thuộc :

cos2 Acos2Bcos2C2cosAcosBcosC 1  5

Chưa nói đến việc phát  5 , riêng việc chứng minh làm “nhức

óc” bạn trẻ làm quen với lượng giác Qua vài ví dụ đây, hẳn bạn thấy vai trị hình học việc phát chứng minh hệ thức “thuần túy lượng giác” Mặt khác, nêu lên cho câu hỏi : Phải hệ thức lượng giác tam giác có “nguồn gốc hình học” làm bạn đường ? Mời bạn giải vài tập sau để củng cố niềm tin

1 Chứng minh rằng, tam giác ta có    

   

2 sin sin sin

2 A B C

R d

trong d khoảng cách đường tròn tâm ngoại tiếp nội tiếp tam giác

(88)

• Cho ABC Dựng mặt phẳng ABC điểm O1 O2 cho

các tam giác O1AB O2AC tam giác cân đỉnh O1,O2 với góc

đáy

30 cho O1 C nửa mặt phẳng bờ AB, O2

và B nửa mặt phẳng bờ AC a) Chứng minh :

OOa b c 3S

6

1 2 2

2

1    

b) Suy bất đẳng thức tương ứng :

sin2 Asin2Bsin2C 2 3sinAsinBsinC

3 Chứng minh ABC có góc nhọn, :

cos cos

cos

sin sin

sin 

 

 

C B

A

C B

A

4 Cho tứ diện OABC có góc tam diện đỉnh O ba mặt vuông,

OC OB

OA  Chứng minh :

sinOABOACcosBAC

(89)

4.4 Từ toán quen thuộc bất đẳng thức

Về toán chứng minh bất đẳng thức gặp nhiều Bái viết này muốn đề cập đến bất đẳng thức quen thuộc lại có thể sử dụng để làm câu nối cho việc chứng minh bất đẳng thức khác phức tạp nói chung khơng đơn giản

1 (Một bất đẳng thức quen thuộc)

Giả sử a,b,c số dương Chứng minh rằng:

+ +

+ + +

a b c

b c a c a b

3

2 (*) (BĐT Nesbbit)

Giải:

Ta kí hiệu vế trái (*) A biến đổi sau: A+3=a b c 1

a b a c b c

 

     

  

 

Đặt u a b v,  a c t,  b c,ta có A+3=1  1

2 u v t u v t

 

     

 

Bây ta biến đổi tích:

B=u v t 1 v u t u t v

u v t u v u t v t

       

              

       

Áp dụng bất đẳng thức Cơsi, ta B≥9 Từ suy ra:

2

A

Dễ thấy đẳng thức xảy : a b c Đến ta tìm cách khai thác toán

Để ý thấy ta nhân hai vế (*) với a b c ta có bất đẳng thức:

   

2 2

3

2

a b c

a b c a b c

b c a c a b

a b c a b c

b c a c a b

 

       

  

 

 

   

  

Kết dẫn đến phép biến đổi tóa hai sau: 2 Giả sửa,b,c số dương Chứng minh rằng:

2 2

2

a b c a b c

b c a c a b

 

  

(90)

Giải:

Ta thấy từ vế phải (1) thêm điều kiện abc=1 theo bất đẳng thức Cơsi ta có:

3

a  b c abc

Khi vế phải (1) biến đổi từ tính chất bắc cầu bất đẳng thức ta có tốn 3:

3 Giả sử a,b,c số dương thõa mãn điều kiện abc=1 Chứng minh

rằng:

2 2

3

a b c

bcacab  (2)

Giải

Như muốn chứng minh bất đẳng thức (2) ta phải xuất phát từ bất đẳng thức (*), sau thực hai bước sau ta có khẳng định:

- Bước 1: Nhân hai vế (*) với a+b+c biến đổi - Bước 2: Chứng minh bất đẳng thức

2

a b c

Đến đây, từ (2)và từ giả thiết ta biến đổi theo bước: - Bước 1: Bình phương hai vế cua (2)

- Bước 2: Nhân hai vế bất đẳng thức sau bước với

2

- Bước 3: Thay biểu thức 2 2 2 , ,

a b a c b c biểu thức tương ứng:

12 , 12 , 12

c b a

Từ ba bước biến đổi ta dẫn đến toán 4:

4. Giả sử a,b,c số dương thõa mãn điều kiện abc=1 Chứng minh rằng:

              

4 4

2 2 2

1 1

8

2 2

a b c

c a c b c b a b b c a a c a b

b c a c a b

     

     

  

Giải:

(91)

5 Giả sử a,b,c số dương thõa mãn điều kiện abc=1 Chứng minh

rằng:

     

3 3

2 2

3

a bcb acc ab  (3)

Giải:

Dễ thấy bất đẳng thức cần chứng minh tương đương với bất đẳng thức:

     

3 3

1 1

2

a bcb acc ab  (4)

Để chứng minh (4), ta đặt: x 1,y 1,z

a b c

  

Khi mua bất đẳng thức (4) có dạng:

2 2

3

x y z

yzxzxy  (**)

Rõ ràng bất đẳng thức thức nội dung toán (do x,y,z số dương xyz=1) Bởi bất đăng thức chứng minh nhờ sử dụng kết toán

Đê ý thêm chút từ toán 1,2 tương đương toán (**), ta phát biểu tốn dạng “mạnh” hơn:

6 Với  1 x> 0,y> 0,z> 0,đồng thời xyz=1, ta có:

x y z

S

y z x z x y

  

    

  

Giải:

Thật vậy, ta để ý đên hai bất đẳng thức:

3

a b c

abc

  

(5)

3

x y z

S

y z x z x y

   

   (6)

Từ bất đẳng thức (5) (6) áp dụng cho ba số x , y , z

yz xz xy

1 1 , ,

x y z

Ta :

 1 1  

1

3

3

3 2

S x y z

S xyz

    

     

(92)

Dễ dàng kiểm tra lại tính đắn bất đăng thức Chẳng hạn xét:

2 2

2

x y z

S

z y x z x y

  

  

Ta phải chứng minh:

 

2

3

3

x y z

x y z

y z x z x y

S

 

   

    

 

  (***) Nghĩa cần chứng minh hai bất đẳng thức:

2 2

3

x y z x y z x y z

y z x z x y y z x z x y

 

 

      

        (7)

3

x y z x y z

y z x z x y

 

    

    

  (8)

Dễ thấy (7) có dạng :

 

x y z

S x y z

y z x z x y

 

     

  

 

2 2

2

x y z x y z

y z x z x y

 

   

   (9)

Rõ ràng (9) có dạng tốn 2, (9) Mặt khác từ bất đẳng thức:

2

x y z

yzxzxy

Và 1

3

x y z

xyz

   

Suy ra:

3

x y z x y z

y z x z x y

   

  

    

  (10)

Từ (9) (10) ta có khẳng đinh (***)

(93)

4.5 Một số tốn khơng thuộc dạng thơng thường

Khơng dám khẳng định bất đẳng thức dễ bất đẳng thức trong dạng tốn nhiều biến hóa khơn lường Một bất đẳng thức khó với người khó với người khác tiếp xúc nhiều với bất đẳng thức giúp ta làm quyen với Đặc biệt các tốn khơng thuộc dạng thơng thường Sau sơ ví dụ cho ta thấy điều

1.Chứng minh với số tự nhiên n> 1, ta có bất đẳng thức: 1 1 12

4 n

       

    

    >

Giải:

Ta có:

  

2 2

2 2

2 1

2

1

1.3 2.4 3.5 1

2.2 3.3 4.4 2

n n

n n n

n n n

  

   

>

Vậy 1 1 12

4 n

       

    

    >

2. Cho a,b,c số không âm Chứng minh rằng:

2 2 2

+ +

a b ca b c- +b c a- +c a b-

Giải:

Bất đẳng thức cho có tính đối xứng thay đổi a,b,c giảu sử a≥b≥c đó:

2 2 2

+ +

a b ca b c( - ) + ( - ) + ( - )b a c c a b

2 2 2

+ +

a b c -2ab+ 2bc≥0

( )2 2

- + + 2

a b c bc≥0 (*)

Dễ thấy (*) với a,b,c≥0

Bới với giá trị không âm thay đổi, bất đẳng thức luôn 3. Chứng minh với số a,b,c có bất đẳng thức

, , ,

a b c b c a c a b thì cá số có tổng hai số

(94)

Giải:

Dễ thấy từ a b c suy  2

abc hay a b ca b c0 (1) Chứng minh tương tự, ta có:

  

  

0.(2) 0.(3)

b c a b c a

c a b c a b

    

    

Từ (1), (2), (3) suy ra:

a b ca b c b  c a b  c a c  a b c  a b0

  2  2 2

0

a c b b c a c a b

        (4)

Vế trái (4) tích số khơng âm, đẳng thức xảy khi:

0 0

a b c

b c a

c a b

   

    

    

Dẫn đến

a b c

b c a

c a b

  

   

   

4 Chứng minh rằng:

x + y + zx+ -y z + x y- +z + - + +x y z

(Đề thi tuyển sinh lớp 10 chuyên toán – Trường THPT Quốc học Huế) Giải:

Ta có:

x+ -y z + x y- +zx+ - + -y z x y+z = 2 x (1) x+ -y z + - +x y+zx+ - - + +y z x y z = 2 y (2) x y- +z + - +x y+zx y- + - +z x y+z = 2 z (3) Từ (1),(2),(3) suy ra:

2(x+ -y z + x y- +z + - +x y+z)2(x + y + z)

x + y + zx+ -y z + x y- +z + - + +x y z

Nhận xét: Để giải toán ta sử dụng tính chất sau: A + BA+B , đẳng thức xảy AB≥0

(95)

Giải:

Ta kí hiệu: 2 2 2

1 2 1 2

= - - - - .

n n n

b a a a a a a

Ta chứng minh rằng:

bn+1 =bn- 1 với n≥5 (*) Thật vậy:

2 2 2 2

+1 = 1 2 +1 - 1 - 2 - - - +1

n n n n n

b a a a a a a a a

( ) 2 2 2 ( )2

1 2 1 2 1 2 1 2

=a a an a a an - -a -a - -an - a a an- 1

2 2 2

1 2 1 2

=a a an -a -a - -an - 1

=bn - 1

Mặt khác: 2 2 2

5 = 1 2 5 - 1 - 2 - - 5 = 65

b a a a a a a

Áp dụng (*), ta có:

6 5

7

70

= - = 64 = 63

= 0

b b b

b

Nghĩa là: 2 2 2 1 2 70 = 1 + 2 + + 70

a a a a a a

Suy ra:

2 2 2

1 2 70

1 2 70

+ +

= 1 2 - 1

a a a

a a a >

6. Dãy số a a a0, 1, 2, ,an, cho với tất số khơng âm m n (m≥n) có hệ thức:

+ - ( 2 2 )

1

+ = + .

2

m n m n m n

a a a a

Chứng minh a1 = 1,

a1994 +a1995< a1993 +a1996

Giải:

* Thật vậy, với m=n ta có a0 = 0

Giả sử n = 0, ta có: + = 1( 2 + 0)

2

m m m

a a a a

(96)

2 +2 2 ( 2 +4 2 )

1

+ = +

2

n n n

a a a a

Áp dụng (1), ta có:a2 +4n = 4an+2

a2n = 4an

Bởi vậy:

2 +2 2 ( 2 +4 2 ) ( +2 )

1 1

+ = + = 4 + 4

2 2

n n n n n

a a a a a a

= 2(an+2 +an) (2) Mặt khác:

a2 +2n +a2 = 4(an+1 +a1)= 4(an+1+ 1) (3) Từ (2),(3) suy ra:

an+2 = 2an+1-an+ 2 (4) Áp dụng (4), ta tính được:

2 2

3 3

2 4

= = 2 = = 3 = 16 = 4

a a a

2

=

n

a n (5) Ta chứng minh (5) quy nạp:

Rõ ràng với n = 0, n = ta có (5) Giả sử (5) với n=k-1và n=k (k≥1)

Khi ta có:

2 ( )2 ( )2

+1 = 2 - -1+ = 2 - - 1 + = + 1

k k k

a a a k k k

Như (5) với n=k+1, nghĩa với n khơng âm Đến ta tính

2 1993

2 1994

2 1995

2 1996

= 1993 = 1994 = 1995 = 1996

a a a a

Từ có bất đẳng thức:

a1994 +a1995< a1993 +a1996

7. Xét tất hàm số bậc hai: ( ) 2

= ax + +

(97)

Giải:

Do f x( )≥0 với x, nên ta có a> Δ = 2- 4

b ac≤0

Suy ra: c≥

2

4

b a

Mặt khác b> a, nên b a- > 0

Bởi + +

-a b c b a

2

+ + 4

-b a b

a b a

+ +

-a b c

b a( )

2 2

4 + 4 +

4

-a ab b a b a

+ +

-a b c b a

2 2

9 + 6 + 4

a ax x

ax (1)

(với xb a-   b a x)

Mặt khác, theo bất đẳng thức Cơsi ta có:

2 2 2

9 9

3

4 2

a ax x a x a x

ax ax ax

       

(2) Từ (1) (2) suy ra: a b c

b c

   

Đẳng thức xảy

2

2

9 ( i x=b-a)

b c

a

x a vo

      

Suy b=c-=4a

8. Các số dương a,b,c thõa mãn điều kiện abc=1 Chứng minh rằng: 5 ab5 5 bc5 5 ac5

ababbcbcacac

Khi đẳng thức xảy ra?

(Dự tuyển vơ địch Tốn Quốc tế lần thứ 37 - 1996) Giải:

Ta có: 5   2 4

abab aa ba babb

   2 2 2

a ba b a ab b a b

       

Do ab2 0 a,b,c > 0, nên  2 2

0

ab aabb

Suy

5 2 

aba b ab

(98)

 

   

5 2

1

.(1)

ab ab

a b ab a b a b ab

c

ab a b ab a b c a b c

   

  

     

Chứng minh tương tự, ta có:

5

5

.(2) (3)

bc a

a b c b c bc

ac b

a b c a c ac

   

   

Từ (1),(2),(3) ta có khẳng định đẳng thức xảy a=b=c=1

9. Một số có 2000 ước số số tự nhiên Chứng minh số lớn hơn 1000000

Giải:

Tất ước số n phân thành cặp (nếu n khơng số phương), cặp có số bé n, cịn số thứ hai lơn n (nếu d bé n, n

d lớn n ) Như số lượng ước số bé n Khi

đó n>2000, suy n>1000000 10. Có hay khơng bất đẳng thức:

xy< z z> 3

3 0?

xyxyz<

Giải:

Do z>0, nên chia hai vế bất đẳng thức 3

3

xyxyz< cho

z , ta có:

3

3

0

x y xyz

z z z

        

    < (1)

Nếu ta kí hiệu u x,v y

z z

  (1) có dạng: 3

3

uvuv< (1’) uv33uv u  v 1< (2)

(99)

11. Chứng minh với số x,y không âm bất kì, ta có bất đẳng thức: 4 3 3

1

xyxyx  y > x yyx

Giải:

Để chứng minh bất đẳng thức cho , ta chứng minh bất đăng thức sau:

y  y y (1)

1

x> x (2) 3

xxyx y (3) Thật vậy, y1 y4  y3

Cịn 0 y yy3 Từ suy (1)

Mặt khác x1

xx, cịn 0 x 1,

1

x> x Bởi (2)

Lại có x y 0,

xx y, (3) đúng, cịn 0 x y hay

0

x y

Khi (3) có dạng:

3  3 

0

xxyx y x xyx y

Do x0, nên suy ra: 3

0

xyx y

  

x y y x y x

     (*) Dễ thấy (*) đúng, (3)

(100)

Chương :

Bất đẳng thứ c thế nào là hay ? Làm có thể sáng ta ̣o bất đẳng thức ?

Bạn đọc đã làm quen với bất đẳng thức từ THCS Bước đầu các bạn có thể chỉ học các bất đẳng thức kinh điển : Cauchy, Bunhia Cốpxki, Jensen,

Chebyshev, Nesbitt … hay bắt đầu đọc SOS, ABC,…Vậy đã bao giờ bạn tự

hỏi Bất đẳng thức thế nào là hay? Làm có thể sáng tạo bất đẳng

thứ c ? Đó thực sự là những vấn đề thú vi ̣ đáng để quan tâm và bình luận

Sau là một số ý kiến của giáo viên toán, học sinh chuyên toán về vấn đề nà y :

Thầy Nguyễn Văn Hiền (GV chuyên toá n Trường THPT chuyên Lê Quý Đôn, Quảng Trị) :

Bất kỳ bất đẳng thức nào cũng đều có cái hay và cái đe ̣p riêng của nó Đă ̣c biệt những bất đẳng thức vâ ̣n du ̣ng nhiều khía ca ̣nh của cái bất biến bất đẳng thức là bất đẳng thức hay!!!

Thầy An (GV chuyên toá n Trường THPT chuyên Lê Quý Đôn, Quảng Trị) : Từ bất đẳng thức ban đầu mà suy đươ ̣c nhiều bất đẳng thức khác là bất đẳng thức hay!!!

Thái Thị Lan (GV dạy toán Trường THCS chuyên Hiếu Giang, Đông

Hà, Quảng Trị)

(101)

Thầy Trần Phương (GĐ Trung tâm hỗ trợ nghiên cứu và phát triển các sản

phẩm trí tuê ̣, là tác giả nhiều cuốn sách hay về toán học sơ cấp) :

Chứ ng minh bất đẳng thức là công viê ̣c đòi hỏi trí thông minh sáng ta ̣o và sự khéo léo

Phạm Kim Hùng (SV khó a Cử nhân tài – Trường ĐHKHTN – ĐHQGHN, là tác giả cuốn sách “Secrets in Inequalities”(Sáng tạo bất đẳng thức) nổi tiếng) :

Điều khó khăn nhất chúng ta tiếp câ ̣n với bất đẳng thức là sự khẳng ̣nh nó có đúng hay không Thực tế thì giải mô ̣t bài toán mang tính “giả thuyết” là mô ̣t viê ̣c khá ma ̣o hiểm và mất nhiều thời gian, thâ ̣m chí sau những cố gắng vâ ̣y thì kết quả thu được chỉ là mô ̣t phản ví du ̣ chứng minh bất đẳng thứ c sai Nhưng toán ho ̣c thì những điều thế này hoàn toàn rất bình thường và các ba ̣n không cần phải e nga ̣i tự phủ ̣nh

một bài toán mình đă ̣t vâ ̣y cả, vì đó sẽ là bước đầu tiên để ba ̣n sáng tạo đươ ̣c mô ̣t bài toán hay và có ý nghĩa

Lê Dương Trường Giang (HS chuyên toá n khóa 2008 – 2011 Trường

THPT chuyên Lê Quý Đôn, Quảng Trị)

Bất đẳng thứ c là mô ̣t mảng toán rất khó, la ̣i là sân chơi để cho những học sinh giỏi toán thể hiê ̣n lực của mình

Nguyễn Ngọc Phương Linh (HS chuyên toá n khóa 2008 – 2011 Trường THPT chuyên Lê Quý Đôn, Quảng Trị)

Bất đẳng thứ c hay là bất đẳng thức có những phát biểu đe ̣p và cách chứng minh thật đă ̣c sắc, có thể khơi gợi những ho ̣c sinh giỏi toán phát triển và tổng quát bài toán

Trần Tiến Minh (HS chuyên toá n khóa 2008 – 2011 Trường THPT chuyên Lê Quý Đôn, Quảng Trị)

(102)

Nguyễn Hương Cảnh (HS chuyên toán khóa 2008 – 2011 Trường THPT

chuyên Lê Quý Đôn, Quảng Trị)

Lấy ý tưởng từ bất đẳng thức khác (khó!) và phát biểu dưới mô ̣t cách khác sau đã áp du ̣ng mô ̣t số bổ đề.Tất nhiên đó trình đô ̣ phải cao hơn, cách làm phải khó hơn, thế mới là sáng ta ̣o !!!

Trương Hữu Hà Ninh (HS chuyên toá n khóa 2002 – 2005 Trường THPT chuyên Lê Quý Đôn, Quảng Trị)

Bất đẳng thứ c có tính tổng quát, khó, đe ̣p là bất đẳng thức hay!!!

Trương Hữu Đông Hà (HS chuyên toá n khóa 2000 – 2003 Trường THPT chuyên Lê Quý Đôn, Quảng Trị)

Những bất đẳng thức ở da ̣ng tổng quát mà trường hơ ̣p đă ̣c biê ̣t của nó là những bất đẳng thức bản, quen thuô ̣c là bất đẳng thức hay!!!

(103)

Chương : Hướng dẫn giải bài tâ ̣p Bài tập chương 1:

Bài 1:

Ta cã: a3+b3  a2b + ab2  a3  - b3 + a2b + ab2  2a3 +a3  2a3 - b3 + a2b + ab2

 3a3  a3 - b3 +a3+ a2b + ab2

 3a3  ( a3 - b3 ) + (a3+ a2b + ab2)

3a3  (a - b)(a2 + ab +b2) +a(a2 +ab +b2) 3a3  (a2 + ab +b2)(2a - b)

a b

b ab a

a

  

 2

2

Hoàn toàn t-ơng tự cho biểu thức lại, ta có:

2 ,

2 2 2

3

2

a c a ca c

c c

b c bc b

b

       

Khi cộng vế BĐT lại với ta đ-ợc BĐT cần chứng minh

Bài 2:

Sử dụng biến đổi BĐT cô si cho số dương ta có

3 ) (

2

2

3

3 2

2

2

ab a

ab ab a

b a

ab a

b a

a

  

    

Hoàn toàn tương tự ta có BĐT

3 ) ( 2

3 2

2

bc b

c b

b

 

 ;

) ( 2

3 2

2

ca c

a c

c

  

Từ ta cần CM

Bài 3:

Chứng minh tương tự 6, ta đưa BĐT dạng b3 a2 c3 b2 a3 c2 3

Sau sử dụng BĐT cô si

ba23 b(2a1) , cb23 c(2b1) , ac23 a(2c1)

(104)

Bài 4:

Theo BĐT cô si dể thấy

2

) ( 1 ) ( 1 2 2 b ab a b a b a b b a a b a               

Tương tự ta củng có hai BĐT với b c cộng lại

                                      2 1 1 1 2 a ca c c bc b b ab a a c c b b a =

3abcabbcca

Đẳng thức xảy a=b=c=1

Bằng cách tương tự ta có tốn với số

Bài 5:

Tương tự Theo BĐTcơ si ta có:

2 1 ; 1 2 c bc b c b b ab a b a             1 ; 1 2 a da d a d d cd c d

c    

       

Cộng theo vế BĐT lại ta có đièu phải chứng minh

Bài

Cũng phương pháp tương tự ta có 2 1 1 2 2 a a a a a

a        

Làm tương tự với b,c d cộng theo vế ta có

2 1 1 1 1 2 2              d c b a d c b a (đpcm)

Đẳng thức xảy a=b=c=d=1

(105)

2 19 2 8 19 16 19 2 a a c b a a c b

a       

   + 2 19 2 8 19 16 19 2 b b a c c b a c

b       

   2 19 2 8 19 16 19 2 c c b a c c b a

c       

   ) ( 2 8

29 a b c

c c b b a a

S   

                        +                                 2 2 2 2 2 c c b b a a c c c b b b a a a             2 2 ) ( 2 2 2

2 c a b c a a a

c b b a a              2 2 2 2

2 b b b c c c a a b b c c

              2 ) (

15 a b c a a b b c c

                                2 2 2 2

3 a a b b c c

                      3 3 2 ) (

15 a b c a a b b c c a b c

29 12 15 2

15   

                               

a a b b c c

Vậy S3.29/ 293 29 Dấu xảy abc2 Bài 8:

Sử dụng bất đẳng thức Schur ta có :

a6 + b6 + c6 + 3a2b2c2 ≥ a4(b2 + c2) +b4(c2 + a2) + c4(a2 + b2) (1) Theo bất đẳng thức AM-GM :

(106)

Bài tập chương 2:

Bài 1:

Ta có :

S C B A c b a cot cot cot 2     

nên bất đẳng thứ c đã cho tương đương với :  1

2 tan tan tan 64 2 C B A c b a S

Mặt khác ta cũng có : sin cos 2 cos 2 2 2 A bc a A bc bc a A bc c b a        

bc A S A A bc A a sin 2 tan sin tan 2    

Tương tự ta cũng có :

S C c S B b tan ; tan 2  

 1 đú ng đpcm

Bài 2:

Ta có :

  A B C

C B A C B A R S p S r C B A S C B A R S abc R sin sin sin sin sin sin sin sin sin sin sin sin sin sin sin          

Vậy : C B A C B A C B A S C B A S r R sin sin sin sin sin sin sin sin sin 2 sin sin sin 2      

Theo Cauchy ta có :

 

sin sin sin sin sin sin sin sin sin

3 A B C A B C

(107)

O A

B C

8 3 sin sin sin

2 3 sin sin

sin

  

C B A

C B

A

 

 

R r 3 S S S

4

3 27

4 đpcm

Bài 3:

Gọi O, G lần lươ ̣t là tâm đường tròn ngoa ̣i tiếp và tro ̣ng tâm ABC

Ta có : OAOBOC3OG

Hiển nhiên :

 

     

 

 

2

cos

cos

cos

0 cos

cos

cos

3

0 ,

cos ,

cos ,

cos

0

2

2

2

  

 

 

 

 

 

 

C B

A

B A

C R

R

OA OC OC

OB OB

OA R

R

OC OB OA

đpcm

Đẳng thứ c xảy OAOBOC0OG0OGABCđều

Bài 4:

Đặt , , , , ,

2 2 2

y z x

axy byz czx

     

   

3

,

2

a b cx y z x y z

        

 

Ta có:

Ta ln có a     b c a b c

 2 2 2  

3

(108)

Bài 5:

Đặt  

 

 ,

,

a x p p

b q x q

   

 , 

a b q p p q

    

Áp dụng bất đẳng thức a b ab ta có

2 2

2 2

yxpxpxqxq  qp2qp2

Vây GTNN y qp2qp2

Bài 6:

Vớim n p q, , , R; Trong mặt phẳng toạ độ, ta đặt

 

 

, ,

a m n

b p q

   a bmp n q,  

Từ suy

a = 2

mn ;b = 2

pq ; ab = mp 2 nq2

Ta có áp dụng công thức a b ab từ ta có đpcm

Đối với tốn ngồi cách chứng minh ta cịn chứng minh bất đẳng thức cách bình phương vế sau sử dụng bất đẳng thức Bunhiacơpxki Cách cho kết tương tự

Bài 7:

(109)

1+ 1

1

1  

 

n

n

 (n=2,3….) (2)

Thật cộng vế bất đẳng thức (2) với số hạng

1

n ta có

1+

1

1   

 

n n

n

 (3)

1 1

1

1 2  

         n n n n n n n n

Từ suy đpcm

Bài 8: y x y y x y

x 2( 5) 6 0, ,

(*) 2   2   

Xét tam thức: f(x) x22(y 5)x6y24y 56 Ta có '(y 5)2 6y2 4y 56

6 ) 5

(    2 

y y y y

2 ) (

5    

y y y

Ta thấy :'0,x,y.Suy : f(x)0,x,y

Đẳng thức xảy

         5 y x Bài 9:                       4 3 1 ) ( 1 ) ( ) ( a a a a a a a a a a a a VT                       4 3 ) ( ) ( a a a a a a a a a a a

(110)

  4 4             

a a a a

a a a a

Dấu “=” xảy 

               1 4 a a a a a a a a a a a a Bài 10:                     b a a d c b d a c c b a VT(1) ) )( ( ) ( ) ( ) )( ( ) ( ) ( b a d c d c d b a b a d c b d c c a d a             2 2 2 ) ( ) ( d c b a c cd ab b d c b a c bc ad a              

 (Áp dụng BĐT VI)

2 ) ( ) ( ) ( ) ( 2 2             d c b a d b c a d c b a (đpcm) Bài 11: Ta có: a b c c b a      

  (Vận dụng BĐT I)

Tương tự :

b a c c a b         c a b b a c        

Cộng theo vế BĐT thức ta đpcm

(111)

Trước hết ta tìm số x,y cho:

M c b y a c b a x c

a3 ) (  ) ( 3 ) (  ) 

(

c y yb

a c xb a

x) 3 3 (1 )

1

(       

sử dụng phưong pháp đồng thức ta có :

  

   

 

  

c b a M y

x y

x

3 ,

2 ,

3

3

Bây ta tìm m,n cho:

) 3 2 3 ( ) (

4bm can abc

Tương tự ta tìm m=6,n=2 Đến ta chứng minh (2) sau Ta có (2) tương đương với

16

2

3

    

 

        

a c

b c

b a c b

a c a

tức

16

1

) 3

(  

  

 

     

a c c b b a c b a

VP tương đương với

       

  1 1  16

       

     

a c a c c b b a a c a c c b b a

(đúng) =>đpcm

Bài 13:

Dễ thấy x1,x2 0

Ta có:

2

1 2

2

2

x x x x

x x x x

   

   

(112)

2

1

2

2

1

1

5

5

x x

x x

x x

x x

 

    

 

 

dễ thấy

2

1

1 x x

x x

0

 , ta có:

2

1

1 x x

x x

  2

1 2

1

2

5 x x x x

x x

 

   (1)

Theo định lí Viét, từ (1) ta có:

2 ( )

2

4

2

5

k

k k

  

  

  

Nghĩa k 52hoặc k  52thì ta có:

2

1

2

3

x x

x x

   

 

   

(113)

Bài tập chương 3:

Bài 1:

Chứ ng minh các bất đẳng thức sau rồi xét dấu bằng xảy :

a

4 cos cos cos

cos cos

cosA BB CC A

b sin2Asin2Bsin2CsinAsinBsinC

c A B C

C B

A 2tan tan tan

1

3

sin

sin

sin

1    

d.

2 tan tan tan cot

cot cot

2 2

2 2

C B A

c b a C

B A

c b

a

   

 

 

 

Bài 1:

Vế trái số nguyên, nên ta đặt x  y

5

y số nguyên Ta tính x theo y

6

5

 

 

y x

y x

Thay

6 

y

x vào vế trái, ta  y  y

36 34

*Theo định nghĩa phần nguyên ta viết:

2 16 34

36 16 34

1 36

34 20

   

  

  

y y

y y

Giải ta 2,1 17

1

  

y

y nhận giá trị 0,1,2 *Với y=0

3

x Thử lại thấy:

8 17

3

 

17 

(114)

*Vậy y=1

x Thử lại

6

2 12

 

    

*Vậy y=2

3

x Thử lại

18 37

3 28

 

18 37 

   

Phương trình có ba nghiệm ; ;

3

1  xx

x

Bài 2:

Ta chứng minh với số tự nhiên n, ta có bất đẳng thức:

2 1 

n n

n

Thật vậy,

) ( ) (

4n n  n

nên n(n1) 2n1

Do đó: ( nn1)2 2n12 n(n1) 4n2

hay nn1 4n2 (vì n, n+1,4n+2 khơng âm) *Từ kết ta viết:

nn1  4n2

Giả sử số noN thoả mãn:

nono1  4no2

Có nghĩa tồn số tự nhiên mo thoả mãn:

nono1mo 4no2

Bất phương trình kép viết thành:

2

2

) ( ) ( )) ( (

) (

4no no   mono   no no    no

Vì ( mo2 (2no 1))2N nên ( mo2 (2no 1))2no 1 Suy mo2 2(2no 1)

o

m số chẵn chia hết cho 2(2no+1) chia hết cho dư

Do khơng tồn no, mo hay điều giả sử

(115)

Bài 3:

Gọi số thực dương cần tìm x1, x2, x3, x4, x5

Và khơng tính tổng qt, ta giả sử:

5 x

0  xxxx (1) Theo giả thiết ta có:

2

1 ( )

x  xxxx (2)

2

2 (x x x x )

x     (3)

2

3 (x x x x )

x     (4)

2

4 (x x x x )

x     (5)

2

5 (x x x x )

x     (6)

Đặt S=x1+ x2+ x3+ x4+ x5, đẳng thức (1), (2), (3), (4), (5), (6)

viết:

2 5

2 4

2 3

2 2

2 1

) (

) (

) (

) (

) (

x S x

x S x

x S x

x S x

x S x

 

 

 

 

 

Do giả thiết (1) ta có:

5

5

4

3

2

1

1 (S x ) (S x ) (S x ) (S x ) (S x ) x

x            (7)

*Ta có đồng thời x1 x5 (suy từ (1))

1 x

x  (suy từ (7)) Ta suy ra:

5

4

3

2

1

1 (S x ) (S x ) (S x ) (S x ) (S x ) x

x           

do x1= x2= x3=x4=x5

Giải phương trình 1 1)

(Sxx (x1 0) (8)

Ta

16xx (x1 0)(9) Nghiệm (9)

16 1 

x , thử lại ta thấy

Đáp số: x1= x2= x3=x4=x5=

16

(116)

Ta kí hiệu diện tích tam giác MBC, MCA s1, s2, s3

Dễ dàng chứng minh:

1 1

s s s s MA

AA  

 suy

1

1 s

s s s MA

MA  

Tương tự ta có:

6 3

3 2

1 1

1

3

2 1

    

 

     

 

     

 

  

  

 

s s s s s

s s s s

s s s MC

MC MB

MB MA

MA

s s s s MC

MC

s s s s MB

MB

) (si

) (

*Gọi diện tích tam giác ABC S

*Dấu đẳng thức (1) xảy khi:

3

S s s

s   

Từ suy M trọng tâm tam giác ABC

Bài 5:

Gọi s3, s4 diện tích tam giác AOB

và BOC

Dễ chứng minh: s1.s2=s3.s4

a)Giả sử AB//CD, ta chứng minh s3=s4

Xét s1+s2+s3+s4=S (1)

Vì s3=s4 nên s3+s4 =2 s3s4 2 s1s2

Từ (1) ta viết:

S s

s S

s s s

s      2 

2 2

1 ( )

ta suy s1  s2  S

b)Giả sử s1  s2  S ta chứng minh AB//CD

Xét 2

2

1 ) ( )

( ssS hay s1s2 2 s1s2 S

Vì s1.s2=s3.s4 nên ss 2 s sS

D

A B

C s3

S1

s2

s4

(117)

Tổ 4.vip.pro.friendly – lớp 10 Toán 117 dấu đẳng thức xảy s3=s4

*Với s3=s4 ta chứng minh khơng khó khăn:

s1+s3=s1+s4

Hai tam giác ABD ABC có diện tích nhau, suy AB//CD Bài 6:

Kẻ đường cao AH, cắt DE K Đặt AH=h, AK=k

*Ta có:

h k h BC DE S

S

ABC

PDE  

h k BC DE

Do ( 2 )

h k h k S

S

ABC

PDE  

Ta có k, h-k hai số không âm mà k+(h-k)=H không đổi, tính k(h-k) đạt giá trị lớn k=h-k hay

2

h k

Do đó:

4 ) (

2

2  

 

h h

h k h k S

S

ABC PDE

Đó điều phải chứng minh

*DE đường trung bình ∆ABC ∆PDE đạt giá trị lớn

Bài 7:

Gọi E, F, H, K giao điểm hình vẽ

Gọi s1, s2, s3, s4 diện tích

các hình bình hànhAEPH, EDKP, PKCF, HPFD S diện tích ABCD

A

D

B

H P

C E

K k

D

E

A

H B

F C K

P s1

(118)

*Dễ dàng chứng minh s1.s3= s2 s4

*Vì s1.s3= s2 s4 >0 nên ta áp dụng bất đẳng thức côsi, ta được:

4 3 2 s s s s s s s s     Suy s s s

s1

1

S a

s

s     

Do s1,s3 lớn

4 S 16 S s s

Vậy tồn s1 s3 không lớn

4

1 diện tích ABCD

Bài 8:

Ta thấy x=2 nghiệm phương trình

Ta cứng minh ngồi x=2, phương trình khơng nhận nghiệm khác

Thật vậy, phương trình cvho viết:

5

3 2

             

a)Nếu x>2

2 5              x 5              x

Nên

5             

x x

b)Nếu x<2:

*Ta thấy x=1, x=0 không thoả mãn phương trình cho *Với x số nguyên âm, ta đặt x=-y y số nguyên dương

(119)

Bài 9:

*Điều kiện 4x6

Ta thấy x2-10x+27=(x-5)2+22

Mặt khác từ bất đẳng thức

2

2

B A B

A  

   

  

Ta có:

2

) ( ) (

2

4     

   

x  x x x

Hay x4 6x 2

*Ta suy ra: x2-10x+27=2 (1)

4  

x

x (2)

Giải (1) ta x=5

Thay x=5 vào (2) ta thấy giá trị hai vế Vậy phương trình có nghiệm x=5

Bài 10:

Theo giả thiết x, y, z số dương từ (1), (2), (3) ta có x2>1, y2>1, z2>1

*Do tính đối xứng hệ phương trình hốn vị vịng quanh

x z y

x   nên khơng tính tổng qt ta giả sử x số lớn xyz

*Từ (1) suy y=x2-1 (4) *Từ (2) suy z=y2-1 (5)

*Từ (3) suy x=z2-1 (4)

Theo điều giả sử xy nên z2-1x2-1 hay z2x2mà x, y, z số dương, nên zx

Theo điều giả sử xz, mà ta có zx, ta suy x=z Chứng minh tương tự x=y

*Giải phương trinh x2-x=1 hay x2-x-1=0

với x>0 ta nghiệm:

5 1

x

*Hệ phương trình có nghiệm x=y=z=

2 1

(120)

Thử lại ta thấy thích hợp

Bài 11:

a) Điều kiện x0

b) *Phương trình cho viết:

 2

8 ) (

0 2 ( ) 4 (

0 16 16

2

2

 

 

  

  

   

 

x x

x x

x x

x x

x x

) (

Ta ln có: 9x2)2 0,( x 2)2 0

Do (x2)28 x 22 0 tương đương với hệ

 

   

 

 

0

0 ) (

2

x x

) (

) (

*Giải (2) ta x=2

Thay x=2 vào (3) ta thấy nghiệm Ta lại thấy 2>0 thoả mãn điều kiện x0

Phương trình có nghiệm x=2 c) Điều kiện x0 Ta có:

) ( 2

2

2    

x x

x

Mà (x 2)2 0 với x không âm Phương trình cho vơ nghiệm

Bài 12:

Ta thấy x=y=x=0 nghiệm hệ

*Khi ba ẩn số x, y ,z nhận giá trị khác giá trị cịn lại khác nữa, chúng lớn

*Ta có: xyz

z y

x

z y x

  

 )(1 )(1 )

(

8

2

2

2 2

Hay 8xyz(1x2)(1 y2)(1z2)

(121)

; ;

            

 

 

      

z z y

y x

x

Dấu đẳng thức xảy x=1 ; y=1 ; z=1

Thẻ lại ta thấy x=y=z=1 nghiệm phương trình cho Vậy hệ phương trình có hai nghiệm (x, y, z)=(0, 0, 0) (x, y, z)= (1, 1, 1)

Bài 13:

Điều kiện -1≤x≤1

*Theo bất đẳng thức Cơsi ta có:

2 1 ) ( 1

2 1 ) ( 1

2 1

) )( (

4

4

4

x x

x

x x

x

x x

x x x

     

     

       

) (

) (

) (

*Từ (1), (2), (3) suy ra:

x x

x x

x        

 1 1

1 4

4

2 1 1

1      

x x

*Dấu bất đẳng thức xảy khi:

1

1

1

 

 

  

x x

x x

) (

) (

) (

Giải hệ phương trình ta x=0

(122)

Mục lục

Chương : Các bước đầu sở ……… 05

1.1 Các bất đẳng thức đại số ……… 06

1.1.1 Bất đẳng thức Cauchy ………06

1.1.1.1 Kĩ thuật chọn điểm rơi BĐT Cauchy ……… 09

1.1.1.2 Kĩ thuật Cauchy ngược dấu ……… 14

1.1.2 Bất đẳng thức BunhiaCốpxki ……… 17

1.1.2.1 Kĩ thuật chọn điểm rơi BĐT BunhiaCốpxki …… 20

1.1.3 Bất đẳng thức Jensen ……… 25

1.2 Các đẳng thức, bất đẳng thức tam giác ……… 27

1.2.1 Đẳng thứ c ……… 27

1.2.2 Bất đẳng thứ c ……… 29

1.3 Bất đẳng thức đối xứng ba biến ……… 30

1.3.1 Bất đẳng thức khơng có điều kiện ……… 30

1.3.2 Bất đẳng thức có điều kiện ……… 31

1.4 Bài tâ ̣p ……… 33

Chương : Các phương pháp chứng minh ……… 34

2.1 Biến đởi tương đương, tính chất bất đẳng thức … 35 2.2 Sử du ̣ng các bước đầu sở ………38

2.3 Đưa về vector và tích vô hướng ……… 41

2.4 Phương pháp quy nạp ……… 45

2.5 Phương pháp phản chứng ……… 47

2.6 Phương pháp dùng tam thức bậc hai ………49

2.7 Sử dụng số bất đẳng thức phụ ……… 52

2.8 Sử dụng định lí Viét ……… 58

2.9 Bài tâ ̣p ……… 61

Chương : Áp du ̣ng vào mô ̣t số vấn đề khác ……… 63

3.1 Định tính tam giác ……… 64

3.1.1 Tam giác ……… 64

3.1.2 Tam giác cân ……… 67

3.1.3 Tam giác vuông ……… 70

3.2 Vận dụng bất đẳng thức vào giải toán ……… 71

3.3 Bài tập ……… 74

Chương : Mô ̣t số chuyên đề bài viết hay, thú vi ̣ liên quan đến bất đẳng thứ c ……… 76

4.1 Về bất đẳng thức có nhiều cách chứng minh ………… 77

(123)

Chương : Bất đẳng thứ c thế nào là hay ?

Làm có thể sáng ta ̣o bất đẳng thức ? ……… 100

Chương : Hướ ng dẫn giải bài tâ ̣p ……… 103

(124)

www.diendan3t.net www.mathnfriend.net www.truongtructuyen.vn

www.nxbgd.com.vn/toanhoctuoitre www.diendantoanhoc.net

www.toanthpt.net

Tạp chí Toán ho ̣c & Tuổi trẻ (NXB Giáo du ̣c) Tạp chí Toán tuổi thơ (NXB giáo du ̣c)

Tuyển tập năm ta ̣p chí Toán ho ̣c & Tuổi trẻ (NXB giáo du ̣c) Tuyển tập 30 năm ta ̣p chí Toán ho ̣c & Tuổi trẻ (NXB giáo du ̣c) Tuyển tập Olympiad 30/4

Tuyển tập các chuyên đề luyê ̣n thi đa ̣i ho ̣c môn Toán – Hê ̣ thức lượng giác (Trần Phương)

10000 bài toán sơ cấp – Tâ ̣p : Bất đẳng thức đa ̣i số (Phan Huy Khải) 10000 bài toán sơ cấp – Tâ ̣p : Bất đẳng thức hình ho ̣c (Phan Huy Khải) Bất đẳng thứ c hình ho ̣c (Vũ Đình Hòa)

Các phương pháp chứng minh bất đẳng thức (Trần Tuấn Anh) Sáng ta ̣o bất đẳng thức (Pha ̣m Kim Hùng)

23 phương pháp chuyên đề bất đẳng thức và cực tri ̣ lươ ̣ng giác (Nguyễn Đức Đồng – Nguyễn Văn Vĩnh)

Tuyển tập 200 bài thi vô ̣nh toán – Tâ ̣p : Lượng giác Toán nâng cao lươ ̣ng giác THPT (Phan Huy Khải)

263 bài toán bất đẳng thức cho ̣n lo ̣c (Nguyễn Vũ Thanh) Bất đẳng thứ c và mô ̣t số vấn đề liên quan (Nguyễn Văn Mâ ̣u)

Phương pháp giải toán bất đẳng thức cưc tri ̣ (Nguyễn Văn Nho – Nguyễn Tiến Dũng – Nguyễn Viê ̣t Hà)

Phương pháp giải 555 bài toán bất đẳng thức đa ̣i số (Nguyễn Đức Đồ ng – Nguyễn Văn Vĩnh)

bất đẳng thức. vế trái tam thức bậc hai dấu tam thức bậc hai phương trình www.truongtructuyen.vn

Ngày đăng: 09/02/2021, 01:56

Từ khóa liên quan

Tài liệu cùng người dùng

Tài liệu liên quan